ChaseDream
标题: 【每日逻辑练习第二季】【1-20】 [打印本页]
作者: fox0923 时间: 2011-12-8 23:42
标题: 【每日逻辑练习第二季】【1-20】
逻辑小分队迎来了第一个系列的最后一贴~~下面等待大家的就是入群选拔拉,大家等候好消息吧~~希望大家在今后的2,3,4.....系列继续跟进和我们一起练习击垮CR~~
--------------------------------狐狐分界线-----------------------------------------
结束了Weaken CR的练习,大家应该都收获了不少吧,接下来再接再厉,上我们Strengthen的题目~加油加油!!
精练
1. Advertisement: At most jewelry stores, the person
assessing the diamond is the person selling it so
you can see why an assessor might say that a
diamond is of higher quality than it really is.
But because all diamonds sold at Gem World
are certified in writing, you’re assured of a fair
price when purchasing a diamond from Gem
World.
The reasoning in the advertisement would be most
strengthened if which one of the following were true?
(A) Many jewelry stores other than Gem World also
provide written certification of the quality of
their diamonds.
(B) The certifications of diamonds at Gem World
are written by people with years of
experience in appraising gems.
(C) The diamonds sold at Gem World are generally
of higher quality than those sold at other
jewelry stores.
(D) The diamond market is so volatile that prices of
the most expensive diamonds can change by
hundreds of dollars from one day to the next.
(E) The written certifications of diamonds at Gem
World are provided by an independent
company of gem specialists.
PREP CR 2 逻辑练习
5. (24305-!-item-!-188;#058&001204)
Microscopic plants called algae grow inside the top layer of sea ice in the Antarctic if enough sunlight reaches that layer of ice and enough seawater, which contains nutrients, reaches that layer after splashing onto the surface. Even though moderate snow cover reduces the sunlight that filters into the top layer, sea ice with moderate snow cover typically contains even more algae in the top layer than does sea ice with less snow cover.
Which of the following, if true, most helps to explain the apparent discrepancy?
(A) As the weight of accumulated snow forces ice lower in the water, more seawater splashes onto the surface and seeps through the snow.
(B) Seawater in the Antarctic often contains higher concentrations of nutrients than does seawater in more temperate regions.
(C) As the air temperature around sea ice decreases during the winter, the likelihood decreases that snow will fall and thus add to any existing snow cover.
(D) The nutrients on which algae rely are common in seawater throughout the Antarctic.
(E) More sunlight filters through a layer of ice than through an equally thick layer of snow.
6. (24353-!-item-!-188;#058&001207)
Which of the following most logically completes the passage?
Garnet and RenCo each provide health care for their employees. Garnet pays for both testing of its employees' cholesterol levels and treatment of high cholesterol. This policy saves Garnet money, since high cholesterol left untreated for many years leads to conditions that require very expensive treatment. However, RenCo does not have the same financial incentive to adopt such a policy, because __________.
(A) early treatment of high cholesterol does not entirely eliminate the possibility of a stroke later in life
(B) the mass media regularly feature stories encouraging people to maintain diets that are low in cholesterol
(C) RenCo has significantly more employees than Garnet has
(D) RenCo's employees are unlikely to have higher cholesterol levels than Garnet's employees
(E) the average length of time an employee stays with RenCo is less than it is with Garnet
7. (24401-!-item-!-188;#058&001291)
United Lumber will use trees from its forests for two products. The tree trunks will be used for lumber and the branches converted into wood chips to make fiberboard. The cost of this conversion would be the same whether done at the logging site, where the trees are debranched, or at United's factory. However, wood chips occupy less than half the volume of the branches from which they are made.
The information given, if accurate, most strongly supports which of the following?
(A) Converting the branches into wood chips at the logging site would require transporting a fully assembled wood-chipping machine to and from the site.
(B) It would be more economical to debranch the trees at the factory where the fiberboard is manufactured.
(C) The debranching of trees and the conversion of the branches into chips are the only stages in the processing of branches that it would be in United's economic advantage to perform at the logging site.
(D) Transportation costs from the logging site to the factory that are determined by volume of cargo would be lower if the conversion into chips is done at the logging site rather than at the factory.
(E) In the wood-processing industry, branches are used only for the production of wood chips for fiberboard.
8. (24449-!-item-!-188;#058&001338)
Which of the following most logically completes the argument?
Utrania was formerly a major petroleum exporter, but in recent decades economic stagnation and restrictive regulations inhibited investment in new oil fields. In consequence, Utranian oil exports dropped steadily as old fields became depleted. Utrania's currently improving economic situation, together with less-restrictive regulations, will undoubtedly result in the rapid development of new fields. However, it would be premature to conclude that the rapid development of new fields will result in higher oil exports, because __________.
(A) the price of oil is expected to remain relatively stable over the next several years
(B) the improvement in the economic situation in Utrania is expected to result in a dramatic increase in the proportion of Utranians who own automobiles
(C) most of the investment in new oil fields in Utrania is expected to come from foreign sources
(D) new technology is available to recover oil from old oil fields formerly regarded as depleted
(E) many of the new oil fields in Utrania are likely to be as productive as those that were developed during the period when Utrania was a major oil exporter
作者: fox0923 时间: 2011-12-8 23:48
今天是bible的例题分析,解释的非常清楚,大家要仔细看啊~
The stimulus is prefaced by the word “advertisement.” One quirk of the LSAT
is that every stimulus in LSAT history that has been preceded by this word has
contained faulty or deceptive logic. Thus, whenever you see this word
prefacing a stimulus, be on the lookout for misleading or flawed reasoning.
The argument is constructed as follows:
Premise: At most jewelry stores, the person assessing the diamond is
the person selling it.
Premise/Sub-conclusion:
So you can see why an assessor might say that a diamond is
of higher quality than it really is.
Premise: All diamonds sold at Gem World are certified in writing,
Conclusion: You’re assured of a fair price when purchasing a diamond
from Gem World.
The first sentence contains a premise and conclusion that relies on the
assumption that financial motivation might cause a person to lie about the
quality of the item. According to the advertisement, at Gem World there is no
such worry because the diamonds are certified in writing. Think for a
moment—does that reasoning sound bulletproof? If you were standing there in
the store and you were told that Gem World has written certification, wouldn’t
you ask who does the certification? This is the essence of personalizing the
argument—place yourself inside the situation and think how you would react.
As soon as you do that in this question, the weakness in the argument becomes
apparent. Then, since this is a Strengthen question, you can look for an answer
choice that eliminates this weakness. Answer choice (E) addresses the hole in
the argument by indicating that the individuals who provide the written
certification are not the same people who are selling the diamonds at Gem
World.
There are other errors in the stimulus, such as assuming that a written
certification equals a fair price. The certification may have no impact on the
actual price of the diamond, or perhaps it could even be used to raise the price
unjustly. These problems are ignored by the answer choices, and the test makers
have that right.
Answer choice (A): The conclusion addresses the fair price of diamonds at
Gem World, not other stores. Hence, the fact that other stores have written
certification does not help the Gem World advertisement.
Answer choice (B): This is an answer many people keep as a Contender. The
answer is incorrect because it fails to address the point raised in the first
sentence, namely that the person assessing the diamond has a personal stake in
the outcome. This “accountability” issue is the central point of the argument,
and without knowing the source of the certifications, this answer does not
strengthen the argument.
Answer choice (C): The argument asserts that a fair price is assured when
purchasing a diamond at Gem World. No claim to comparative quality is made
in the advertisement, and thus this answer does not strengthen the argument.
Answer choice (D): If anything, this answer may hurt the argument since it
indicates that a fair price may not be obtainable at Gem World due to price
volatility. If prices change daily, then Gem World may be selling diamonds at a
price that does not reflect current market value. However, the answer choice
specifically mentions “the most expensive diamonds” and there is no guarantee
that Gem World carries diamonds in this price range. So, at best, the answer
choice has no effect on the argument and is therefore incorrect.
Answer choice (E): This is the correct answer. As mentioned above, this answer
addresses the separation of the certification writer from the seller and thereby
strengthens the reasoning.
PREP CR KEY: AEDB
作者: daisyの小夢想 时间: 2011-12-9 00:05
哇呼呼。时间过得真快,期待勤奋的战友们入群~
作者: fox0923 时间: 2011-12-9 00:31
精练----------------------31s---------------------------strengthenP: The person who assess the diamond is the one sell it to you, we could know that these people value the diamond higher than it really is.
P: All the sold diamonds are certified in writing.
C: So the price of diamond will be fair for us.
Prephrase: People who write the certification of these diamonds in Gem world are not people who assess the value of diamonds.
Analysis:
(A) Many jewelry stores other than Gem World also
provide written certification of the quality of
their diamonds.-------------------------------------------------We don't care about other stores, we care if the value and price of diamonds are fair.
(B) The certifications of diamonds at Gem World
are written by people with years of
experience in appraising gems.------------------------------this can be the contender to eliminate our doubt. 我给选错了,在B/E中徘徊,然后B实际上是重复了premise,虽然加了year of experience,但是不能说明这些人就qualify了。
(C) The diamonds sold at Gem World are generally
of higher quality than those sold at other
jewelry stores.--------------------------------------------------higher quality doesn't mean fair price.
(D) The diamond market is so volatile that prices of
the most expensive diamonds can change by
hundreds of dollars from one day to the next.------------weaken the conclusion somewhat.
(E) The written certifications of diamonds at Gem
World are provided by an independent
company of gem specialists.---------------------------------whether the company or person writes this certification is not important, the issue is whether the company or person is qualified to write the certification.As mentioned above, this answer addresses the separation of the certification writer from the seller and thereby strengthens the reasoning.
---------------------------------------------------------------------------------------------------------------------------------------------
小总结一下:
1. 看到了advertisment这类的内容,就要联想到这个argument有flaw;
2. 把自己置身于场景中,然后问自己:If you were standing there in the store and you were told that Gem World has written certification, wouldn’t you ask who does the certification?
这种方法没有尝试过,下次做题的时候试试看。
3. strengthen问题可以去掉有可能weaken argument的choice--〉since this is a Strengthen question, you can look for an answer choice that eliminates this weakness. 这里和我起初想象的事一样的。
4. 其他的prephrase:There are other errors in the stimulus, such as assuming that a written certification equals a fair price.
The certification may have no impact on the actual price of the diamond, or perhaps it could even be used to raise the price unjustly.
我自己肯定想不到这么多~~
5. 改错~~B/E
作者: zz42050524 时间: 2011-12-9 09:33
哇呼呼。时间过得真快,期待勤奋的战友们入群~
-- by 会员 daisyの小夢想 (2011/12/9 0:05:01)
快啦,快啦,马上这一期做完就能入群啦~~~~
作者: zz42050524 时间: 2011-12-9 09:48
2011/12/9 support 33s
At most jewelry stores, people who sell the jewelry assess it, so assessor says that the jewelry is higher the quality than it real is.
People in A store can get the jewelry in its real value.
Pre: 在大多数商店和A商店买珠宝会有不同的结果,前提解释了大多数商店的原因,那么答案是就是Astore和大多数商店估价不同,以导致来了A不同的效果,增强结论。
(A) Many jewelry stores other than Gem World also 这个似乎是一个weaken,其实是无关,前提突出的是
provide written certification of the quality of 估价者是卖者这层有利益关系的评估
their diamonds.
(B) The certifications of diamonds at Gem World 似乎是support,其实没有,没有契合到突出点
are written by people with years of
experience in appraising gems.
(C) The diamonds sold at Gem World are generally irrelevant
of higher quality than those sold at other
jewelry stores.
(D) The diamond market is so volatile that prices of irrelevant
the most expensive diamonds can change by
hundreds of dollars from one day to the next.
(E) The written certifications of diamonds at Gem 这是一个support,是答案,突出了估价者和珠宝的
World are provided by an independent 销售无利益关系。
company of gem specialists.
作者: qiuhua01234567 时间: 2011-12-9 13:09
入群,入群
Background: Manufacturers sometimes discount the price of a product to retailers for a promotion period
when the product is advertised to consumers
Premise: Such promotions often result in a dramatic
increase in amount of product sold by the manufacturers to retailers
conclusion the
manufacturers could often make more profit by not holding the promotions.
(A)The amount of discount generally offered by manufacturers to retailers is carefully
calculated to represent the minimum needed to draw consumers' attention to the product.
(B) For many consumer products the period of advertising discounted prices to consumers is
about a week, not sufficiently long for consumers to become used to the sale price.
(C) For products that are not newly introduced, the purpose of such promotions is to keep
the products in the minds of consumers and to attract consumers who are currently using
competing products.
(D) During such a promotion retailers tend to accumulate in their warehouses inventory
bought at discount; they then sell much of it later at their regular price.
-----------------------------------------------------------------------------------------correct
(E) If a manufacturer fails to offer such promotions but its competitor offers them, that
competitor will tend to attract consumers away from the manufacturer's product.
Background:Researcher: People with certain personality disorders have more theta brain waves than those without such disorders
Premise: my data show that the amount of one’s theta brain waves increases while watching TV
Conclusion: watching too much TV increases one’s risk of developing personality disorders
(A)uses the phrase “personality disorders” ambiguously
(B) fails to define the phrase “theta brain waves”
(C) takes a correlation to imply a causal connection
----------------------------------------------------------------------------correct.
(D) draws a conclusion from an unrepresentative sample of data
(E) infers that watching TV is a consequence of a personality disorder
2.background: Excavation of the ancient city of Kourion on the island
of Cyprus revealed a pattern of debris and collapsed buildings typical of towns devastated by earthquakes.
Conclusion: destruction was due to a major earthquake known to have occurred near the island in A.D. 365
(A)Bronze ceremonial drinking vessels that are often found in graves dating from years preceding and following A.D. 365 were also found in several graves near Kourion.
(B) No coins minted after A.D. 365 were found in Kourion, but coins minted before that year were found in abundance.
---------------------------------------------------------------------------------------------------------correct
(C) Most modern histories of Cyprus
mention that an earthquake occurred near the island in A.D. 365.
(D) Several small statues carved in styles current in Cyprus in the century between A.D. 300 and 400 were found in Kourion.
(E) Stone inscriptions in a form of the Greek alphabet that was definitely used in Cyprus after A.D. 365 were found in Kourion.
Background: According to a review of 61 studies of patients suffering from severely debilitating depression, a large majority of the patients reported that missing a night's sleep immediately lifted their depression
Conclusion: Yet sleep-deprivation is not used to treat depression even though the conventional treatments, which use drugs and electric shocks, often have serious side effects.
(A)For a small percentage of depressed patients, missing a night's sleep induces a temporary sense of euphoria.
(B) Keeping depressed patients awake is more difficult than keeping awake people who are not depressed.
(C) Prolonged loss of sleep can lead to temporary impairment of judgment comparable to that induced by consuming several ounces of alcohol.
(D) The dramatic shifts in mood connected with sleep and wakefulness have not been traced to particular changes in brain chemistry.
(E) Depression returns in full force as soon as the patient sleeps for even a few minutes.
-------------------------------------------------------------------------------------------------------------correct
Background: In many corporations, employees are being replaced by automated equipment in order to save money
Conclusion: many workers who lose their jobs to automation will need government assistance to survive, and the same corporations that are laying people off will eventually pay for that assistance through increased taxes and unemployment insurance payments.
(A)Many workers who have already lost their jobs to automation have been unable to find new jobs.
--------------------------------------------------------------------------------------------------------------correct
(B) Many corporations that have failed to automate have seen their profits decline.
(C) Taxes and unemployment insurance are paid also by corporations that are not automating.
(D) Most of the new jobs created by automation pay less than the jobs eliminated by automation did.
(E) The initial investment in machinery for automation is often greater than the short-term savings in labor costs.
Background: United States
hospitals have traditionally relied primarily on revenues from paying patients to offset losses from unreimbursed care
(A)Although the advance of technology has made expensive medical procedures available to the wealthy, such procedures are out of the reach of low-income patients.
(B) If hospitals do not find ways of raising additional income for unreimbursed care, they must either deny some of that care or suffer losses if they give it.
--------------------------------------------------------------------------------------------------------correct
(C) Some patients have incomes too high for eligibility for governmental health insurance but are unable to afford private insurance for hospital care.
(D) If the hospitals reduce their costs in providing care, insurance companies will maintain the current level of reimbursement, thereby providing more funds for unreimbursed care.
(E) Even though philanthropic donations have traditionally provided some support for the hospitals, such donations are at present declining.
Background:At most jewelry stores, the person assessing the diamond is the person selling it so
you can see why an assessor might say that a diamond is of higher quality than it really is.
Premise:all diamonds sold at Gem World are certified in writing
Conclusion:you’re assured of a fair price when purchasing a diamond from Gem
World.
(A)Many jewelry stores other than Gem World also
provide written certification of the quality of
their diamonds.
(B) The certifications of diamonds at Gem World
are written by people with years of
experience in appraising gems.
(C) The diamonds sold at Gem World are generally
of higher quality than those sold at other
jewelry stores.
(D) The diamond market is so volatile that prices of
the most expensive diamonds can change by
hundreds of dollars from one day to the next.
(E) The written certifications of diamonds at Gem
World are provided by an independent
company of gem specialists.
----------------------------------------------------------correct
Background: Microscopic plants called algae grow inside the top layer of sea ice in the Antarctic if enough sunlight reaches that layer of ice and enough seawater, which contains nutrients, reaches that layer after splashing onto the surface
Conclusion: Even though moderate snow cover reduces the sunlight that filters into the top layer, sea ice with moderate snow cover typically contains even more algae in the top layer than does sea ice with less snow cover.
(A) As the weight of accumulated snow forces ice lower in the water, more seawater splashes onto the surface and seeps through the snow.
-------------------------------------------------------------------------------------------correct
(B) Seawater in the Antarctic often contains higher concentrations of nutrients than does seawater in more temperate regions.
(C) As the air temperature around sea ice decreases during the winter, the likelihood decreases that snow will fall and thus add to any existing snow cover.
(D) The nutrients on which algae rely are common in seawater throughout the Antarctic.
(E) More sunlight filters through a layer of ice than through an equally thick layer of snow.
background: Garnet and RenCo each provide health care for their employees. Garnet pays for both testing of its employees' cholesterol levels and treatment of high cholesterol.
Premise: This policy saves Garnet money, since high cholesterol left untreated for many years leads to conditions that require very expensive treatment
Conclusion: RenCo does not have the same financial incentive to adopt such a policy
(A) early treatment of high cholesterol does not entirely eliminate the possibility of a stroke later in life
(B) the mass media regularly feature stories encouraging people to maintain diets that are low in cholesterol
(C) RenCo has significantly more employees than Garnet has
(D) RenCo's employees are unlikely to have higher cholesterol levels than Garnet's employees
(E) the average length of time an employee stays with RenCo is less than it is with Garnet
-----------------------------------------------------------------------------------------------correct
(A) Converting the branches into wood chips at the logging site would require transporting a fully assembled wood-chipping machine to and from the site.
(B) It would be more economical to debranch the trees at the factory where the fiberboard is manufactured.
(C) The debranching of trees and the conversion of the branches into chips are the only stages in the processing of branches that it would be in United's economic advantage to perform at the logging site.
(D) Transportation costs from the logging site to the factory that are determined by volume of cargo would be lower if the conversion into chips is done at the logging site rather than at the factory.
--------------------------------------------------------------------------------correct
Background: Utrania was formerly a major petroleum exporter, but in recent decades economic stagnation and restrictive regulations inhibited investment in new oil fieldsIn consequence, Utranian oil exports dropped steadily as old fields became depleted. Utrania's currently improving economic situation, together with less-restrictive regulations, will undoubtedly result in the rapid development of new fields
Conclusion: it would be premature to conclude that the rapid development of new fields will result in higher oil exports
(A)the price of oil is expected to remain relatively stable over the next several years
(B) the improvement in the economic situation in Utrania is expected to result in a dramatic increase in the proportion of Utranians who own automobiles
(C) most of the investment in new oil fields in Utrania is expected to come from foreign sources
(D) new technology is available to recover oil from old oil fields formerly regarded as depleted
------------------------------------------------------------------------------correct
(E) many of the new oil fields in Utrania are likely to be as productive as those that were developed during the period when Utrania was a major oil exporter
看错,悔
作者: fox0923 时间: 2011-12-9 13:12
哇呼呼。时间过得真快,期待勤奋的战友们入群~
-- by 会员 daisyの小夢想 (2011/12/9 0:05:01)
快啦,快啦,马上这一期做完就能入群啦~~~~
-- by 会员 zz42050524 (2011/12/9 9:33:38)
恩恩,快啦快啦
作者: winghyy 时间: 2011-12-9 15:25
精炼题
45s-strengthen
At most jewelry store, the person who assess the diamond is the person who sell it. That's why an assessor say that a diamond is of higher quaility than it is. At Gem World, all diamond sold are certified in writing, so you can get a fair price.
Prephase: The assessor is from a independent certified authority.
选E
(A) Many jewelry stores other than Gem World also provide written certification of the quality of
their diamonds. ——weaken, Gem world has no advantage.
(B) The certifications of diamonds at Gem World are written by people with years of
experience in appraising gems.——tricky, these people can also be a employee of GEM
(C) The diamonds sold at Gem World are generally of higher quality than those sold at other
jewelry stores.—— irrelevant (high quality)
(D) The diamond market is so volatile that prices of the most expensive diamonds can change by
hundreds of dollars from one day to the next.——irrelevant (the whole market)
(E) The written certifications of diamonds at Gem World are provided by an independent
company of gem specialists. ——bingo
作者: winghyy 时间: 2011-12-9 17:28
逻辑链
5.55s-discrepancy
Algae grow when enough sunlight reaches the layer of ice and enough seawater reach that layer.
Seaice with moderate snow cover contains more algae than with less snow cover, although moderate snow cover reduces the sunlight.
Prephase: moderate snow cover keep a stable temperature, with the same effect of sunlight.
选A
6.46s-complete(because)
G pays for both testing of C levels and treatment of C. This policy help G save money, since the treatment cost is very high.
But R does not adopt this policy, because_______
Prephase: R company buy an health insurance for every employee.
选E
7. 80s-support the following (must be true)
United Lumber use trees for two products. Tree truncks will be used for lumbers and the branch will be converted into wood chips. The cost of the convertion will be the same at the different place. However, wood chips occupy less than half the volumn of the branches.
选A
8. 45s-complete (because)
U is improving economic situation, together with less restrictive regulations. It will result in the rapid development of new fields. However, it would be premature to conclude that the development of new fields will result in higher oil exports,because____
Prephase: The oil drilled from the new field can not meet the needs of local market.
选B
作者: daisyの小夢想 时间: 2011-12-9 21:42
做满20次练习的XDJM们直接加群191147967啦哈哈。
验证注明你在CD的昵称我就知道了。
欢迎欢迎^^
作者: xeyyxzty 时间: 2011-12-9 21:45
1. background information:At most jewelry stores, the person assessing the diamond is the person selling it so you can see why an assessor might say that a diamond is of higher quality than it really is.premise: all diamonds sold at Gem World are certified in writing,
conclusion:you’re assured of a fair price when purchasing a diamond from Gem World.
(A) Many jewelry stores other than Gem World also provide written certification of the quality of their diamonds.
--irrelevant
(B) The certifications of diamonds at Gem World are written by people with years of experience in appraising gems.
--weaken
(C) The diamonds sold at Gem World are generally of higher quality than those sold at other jewelry stores.
--irrelevant
(D) The diamond market is so volatile that prices of the most expensive diamonds can change by hundreds of dollars from one day to the next.
--irrelevant
(E) The written certifications of diamonds at Gem World are provided by an independent company of gem specialists.
--right
2.background information:Microscopic plants called algae grow inside the top layer of sea ice in the Antarctic if enough sunlight reaches that layer of ice and enough seawater, which contains nutrients, reaches that layer after splashing onto the surface.
premise: moderate snow cover reduces the sunlight that filters into the top layer, +???
conclusion:sea ice with moderate snow cover typically contains even more algae in the top layer than does sea ice with less snow cover.
offer enough nutrients
(A) As the weight of accumulated snow forces ice lower in the water, more seawater splashes onto the surface and seeps through the snow.
--right
(B) Seawater in the Antarctic often contains higher concentrations of nutrients than does seawater in more temperate regions.
--irrelevant
(C) As the air temperature around sea ice decreases during the winter, the likelihood decreases that snow will fall and thus add to any existing snow cover.
--irrelevant
(D) The nutrients on which algae rely are common in seawater throughout the Antarctic.
--irrelevant
(E) More sunlight filters through a layer of ice than through an equally thick layer of snow.
--weaken
3.background information:Garnet and RenCo each provide health care for their employees. Garnet pays for both testing of its employees' cholesterol levels and treatment of high cholesterol.
premise: high cholesterol left untreated for many years leads to conditions that require very expensive treatment
conclusion: This policy saves Garnet money
employees of the G company have a higher risk in getting high cholesterol because of job
(A) early treatment of high cholesterol does not entirely eliminate the possibility of a stroke later in life
--irrelevant
(B) the mass media regularly feature stories encouraging people to maintain diets that are low in cholesterol
--irrelevant
(C) RenCo has significantly more employees than Garnet has
--irrelevant
(D) RenCo's employees are unlikely to have higher cholesterol levels than Garnet's employees
--right
(E) the average length of time an employee stays with RenCo is less than it is with Garnet
--irrelevant--right
思路错了~
4.premise:United Lumber will use trees from its forests for two products. The tree trunks will be used for lumber and the branches converted into wood chips to make fiberboard. The cost of this conversion would be the same whether done at the logging site, where the trees are debranched, or at United's factory. However, wood chips occupy less than half the volume of the branches from which they are made.
conclusion:???
done at the logging site
(A) Converting the branches into wood chips at the logging site would require transporting a fully assembled wood-chipping machine to and from the site.
--irrelevant
(B) It would be more economical to debranch the trees at the factory where the fiberboard is manufactured.
--wrong
(C) The debranching of trees and the conversion of the branches into chips are the only stages in the processing of branches that it would be in United's economic advantage to perform at the logging site.
--unknown
(D) Transportation costs from the logging site to the factory that are determined by volume of cargo would be lower if the conversion into chips is done at the logging site rather than at the factory.
--right
(E) In the wood-processing industry, branches are used only for the production of wood chips for fiberboard.
--irrelevant
5.background information:Utrania was formerly a major petroleum exporter, but in recent decades economic stagnation and restrictive regulations inhibited investment in new oil fields. -->Utranian oil exports dropped steadily as old fields became depleted.
Utrania's currently improving economic situation, together with less-restrictive regulations, will undoubtedly result in the rapid development of new fields.
premise:???
conclusion: premature to conclude that the rapid development of new fields will result in higher oil exports
foreign countries have less demand for importing petroleum
(A) the price of oil is expected to remain relatively stable over the next several years
--irrelevant
(B) the improvement in the economic situation in Utrania is expected to result in a dramatic increase in the proportion of Utranians who own automobiles
--right
(C) most of the investment in new oil fields in Utrania is expected to come from foreign sources
--irrelevant
(D) new technology is available to recover oil from old oil fields formerly regarded as depleted
--irrelevant
(E) many of the new oil fields in Utrania are likely to be as productive as those that were developed during the period when Utrania was a major oil exporter
--weaken
作者: fox0923 时间: 2011-12-9 21:47
祝贺一下我们有新成员的加入了~~
作者: 蘑菇胖肉肉 时间: 2011-12-10 00:03
糊糊糊糊话说我不怎么懂这个队伍诶。。。怎么玩呀
作者: balapupu 时间: 2011-12-10 11:16
P:[25s]
P: In most cases, people want to see diamond to make sure whether the diamond quality is exaggerated by the seller? because all the diamond in Gen has certificate?C: the diamond has a fair price.
Support: as to the same quality diamond, the certificate diamond price will not higher than the non certificate price.
A: (A) Many jewelry stores other than Gem World also
provide written certification of the quality of
their diamonds.—>irrelevant with other jewelry stores.
(B) The certifications of diamonds at Gem World
are written by people with years of
experience in appraising gems.--> experience can not guarantee the fairness.
(C) The diamonds sold at Gem World are generally
of higher quality than those sold at other
jewelry stores.-->irrelevant with comparison of the quality
(D) The diamond market is so volatile that prices of
the most expensive diamonds can change by
hundreds of dollars from one day to the next.-->weaken
(E) The written certifications of diamonds at Gem
World are provided by an independent
company of gem specialists.-->R
作者: fox0923 时间: 2011-12-10 12:05
糊糊糊糊话说我不怎么懂这个队伍诶。。。怎么玩呀
-- by 会员 蘑菇胖肉肉 (2011/12/10 0:03:08)
嘿嘿~就是直接跟贴和我们一起做bible CR精练题目和PREP CR题目就可以了,每天坚持住就好了,然后20天算是一个系列,我们会将坚持每天练习的同学加入到逻辑群里方便更好的讨论。蘑菇快点一起跟做吧~~~
作者: 风无衣 时间: 2011-12-10 13:18
【精炼1-20】
strengthen_41s
background information:people who assess the diamond is the person who sell it. you can see why an assessor will say that a diamond is of higher quality than it really is.
premise:in Gem World, all diamonds are assessing in writing.
conclusion:you can buy diamond in Gem World at a fair price.
(A) Many jewelry stores other than Gem World also
provide written certification of the quality of
their diamonds.——irrelevant
(B) The certifications of diamonds at Gem World
are written by people with years of
experience in appraising gems.——it is possible that when people with years of experience in appraising gems write the certifications, they will intentionally write the certification that the diamond is of higher quality than it really is.
(C) The diamonds sold at Gem World are generally
of higher quality than those sold at other
jewelry stores.——has no effect on this issue
(D) The diamond market is so volatile that prices of
the most expensive diamonds can change by
hundreds of dollars from one day to the next.——irrelevant
(E) The written certifications of diamonds at Gem
World are provided by an independent
company of gem specialists.——correct
The stimulus is prefaced by the word “advertisement.” One quirk of the LSAT
is that every stimulus in LSAT history that has been preceded by this word has
contained faulty or deceptive logic. Thus, whenever you see this word
prefacing a stimulus, be on the lookout for misleading or flawed reasoning.
作者: ugly5552000 时间: 2011-12-15 20:50
1/Background: in most jewelry stores, because the assessors of diamonds is also the sellers of diamonds, they often exaggerate the quality of the diamonds they sell.
Premise: every diamond from Gem World is certified in writing, so the price of diamonds are faire to clients.
Prephrase: the certification of diamonds is provided by the neutral institute.
E
A: if it is true, the Gem World’s written certification might be unreliable as others, it weakens the argument.
B: what if the people who write the certification are the shop’s sellers?
C: irrelevant
D: irrelevant
E: right, the certification provided by an independent company is convincible.
2/Background: enough sunshine and seawater help algae to grow inside the top layer of sea ice. Sea ice with moderate snow cover contains more algae than does sea ice with less snow cover.
Prephrase: the moderate snow cover makes sea ice sink deeper, which makes it easy for seawater to reach the layer where algae grows.
A
A: it meets one of the requirements which promote the growth of algae.
B: irrelevant
C: irrelevant
D: irrelevant
E: it weakens the argument
3/Background: Garnet and RenCo each provide health care for their employees. Garnet’s policy of testing and curing high cholesterol saves its money, for high cholesterol left untreated for years leads to conditions that require very expensive treatment. RenCo does not have the same financial incentive to adopt such a policy.
Prephrase: RenCo is in a trade different from Garnet, it is much less likely for its employees to catch disease, or the average less of time employees work in RenCo is less than 2 years.
E
A: irrelevant
B: irrelevant
C: it cannot be the explanation
D: compared with E, it is not the most suitable answer.
E: the employees often leave the company before they are likely to catch high cholesterol.
4/Background: the tree trunks are used for lumber and the branches are converted into wood chips. To do the convention at logging site or at factory does not affect the cost. Wood chips cost less than half the volume of the branches from which they are made.
Prephrase: the convention will be transferred into the factory, it is easy to tackle with the remnant of wood after the convention of branches.
E
A: a fully assembled wood-chipping machine is not referred to in the main part.
B: if it is true, the cost of debranch will increase.
C: not mentioned
D: a truck can transport more chips than branched each time.
E: not mentioned
5/Background: because of restrictive regulation, Utranian oil exports dropped steadily as old fields became depleted.
Premise: Utrania’s currently improving economic situation, together with less-restrictive regulations, will result in the rapid development of new fields, but it is not certain that the exports will increase as well.
Prephrase: to meet the requirements of domestic demand is the first step. The oversea clients have found substitute for Utrania.
B
A: here the price has no ties with the amount of export.
B: when there is a substantial increase of automobiles, the Utrania should meet the domestic demand of oil
C: irrelevant
D: irrelevant
E: irrelevant
作者: leewonting 时间: 2012-4-17 22:32
102
背景:At most jewelry stories, the person assessing the diamond is the person selling it so you can see why an assessor might say that a diamond is of higher qualified than it really is.
条件:The diamond in Gem World is certified in writing.
结论:Fair price.
推测:有certified的钻石不能乱提高价格?
选E(B is confusing)
103
背景:Microscopic plants called algae grow inside the top layer of sea ice in the Antarctic if enough sunlight reaches that layer of ice and enough seawater which contains nutrients.
条件:Moderate snow cover reduces the sunlight that filters into the top layer.
结论:But sea ice with moderate snow cover typically contains more algae in the top layer than does sea ice with less snow cover
推测:snow cover contains more nutrients?
选A
104
背景:Garnet and RenCo each provide health care for their employees.
条件:Garnet provides both testing and treatment of high cholesterol, but RenCo does not.
结论:RenCo does not have such these incentives
推测:RenCo hires people quitting their jobs from Garnet
选E
105
背景:The tree trunks will be used for lumber and the branches converted into wood chips to make fiberboard.
条件:Cost of this conversion would be the same whether done at the logging site or at United’s factory
结论:Wood chips occupy less than half the volume of the branches from which they are made.
推测:wood chips的制作地点是不是要交通费用?
选D
106
背景:Utrania was formerly a major petroleum exporter, but in recent years economic stagnation and restrictive regulations inhibited investment in new oil fields.
条件:Old fields became depleted and there are no restrictive regulations
结论:It’s premature to conclude that the rapid development of new fields will result in higher oil export.
推测:国内对oil的需求也很大,用于内需
选B
作者: FB小贝 时间: 2012-5-15 22:38
1. Advertisement: At most jewelry stores, the person
assessing the diamond is the person selling it so
you can see why an assessor might say that a
diamond is of higher quality than it really is.
But because all diamonds sold at Gem World
are certified in writing, you’re assured of a fair
price when purchasing a diamond from Gem
World.
The reasoning in the advertisement would be most
strengthened if which one of the following were true?
(A) Many jewelry stores other than Gem World also
provide written certification of the quality of
their diamonds.
(B) The certifications of diamonds at Gem World
are written by people with years of
experience in appraising gems.
(C) The diamonds sold at Gem World are generally
of higher quality than those sold at other
jewelry stores.
(D) The diamond market is so volatile that prices of
the most expensive diamonds can change by
hundreds of dollars from one day to the next.
(E) The written certifications of diamonds at Gem
World are provided by an independent
company of gem specialists.
E
作者: spencerX 时间: 2012-6-1 23:45
【精练】
Background: Because it is who sell the diamonds to you that assesses them, the price of diamonds is higher than their real quality.
Premise: All diamonds sold at GW are certified in writing
Conclusion: You can buy diamonds at GW at a fair price.
Pre: The certifications are not given by who sell the diamonds to customers.
Analysis:
A. Out of scope. What concerned in this argument is weather the certification given by GW is neutral, information about other companies cannot strengthen the argument.
B. Even when the certifications are written by people with years of experience in appraising gems, we cannot know whether they will assess the diamonds neutral.
C. Irrelevant.
D. Irrelevant.
E. Correct. If this statement is true, it is reasonable to infer that customers can buy high-end diamonds at a fair price.
【逻辑链】
1.
Paradox: 阳光照射使长在海冰上的plants生长。但是当moderate雪减少阳光的时候,plants长得更多了。
Pre:雪保存了营养?
My answer:A
2.
Background:G and R each provide health care for their employees.
Premise: G pays: test- c levels& treatment of high c; high cholesterol left untreated for many years leads to conditions that require very expensive treatment
Subordinate conclusion: G’s strategy can save money
Conclusion: R does not have to adapt the same policy
Pre: R’s employees’ c will not last long
My answer: E
3.
Fact:
United Lumber will use trees from its forests for two products.
Tree trunks: lumber
Branches: wood chips to make fiberboard
Cost of the conversion would be the same whether done at the logging site or factory
HOWEVER, wood chips occupy less than half the volume of the branches from which they are made.
Pre: It will cost less money if people do the job at the site.
My answer: D
虽然在哪里处理树,价格都是一样的,但是如果chop占的地方少,而运费又是根据体积来算的,那么在site处理好之后再运走,会比较节约
4.
Background: Economic stagnation& restrictive regulations inhibited the oil export of U. U oil exports dropped steadily as old fields became depleted.
Premise: U will improve economic situation and have less restrictive regulations, as a result, new fields will develop rapidly.
Conclusion: It is premature to get the conclusion--- development of new fields→ higher oil exports
Pre: ?
My answer: B
这题是“他因”,如果U本国的汽车多了,那么就会使用油,则出口量就不一定增加。
作者: baseboss 时间: 2012-6-2 21:18
1.In contract to nomarl diamond assessing by selling guy.G certified the price in write.
presume:
Certified price is the same as the really price.
(A) Many jewelry stores other than Gem World also
provide written certification of the quality of
their diamonds.
-->weaken
(B) The certifications of diamonds at Gem World
are written by people with years of
experience in appraising gems.
-->irrelevant
(C) The diamonds sold at Gem World are generally
of higher quality than those sold at other
jewelry stores.
-->irrelevant
(D) The diamond market is so volatile that prices of
the most expensive diamonds can change by
hundreds of dollars from one day to the next.
-->weaken
(E) The written certifications of diamonds at Gem
World are provided by an independent
company of gem specialists.
-->support,correct
2 presume:
moderate snow cover can hold more nutrients when seawater splashing on to it.
(A) As the weight of accumulated snow forces ice lower in the water, more seawater splashes onto the surface and seeps through the snow.
-->correct
(B) Seawater in the Antarctic often contains higher concentrations of nutrients than does seawater in more temperate regions.
-->irrelevant
(C) As the air temperature around sea ice decreases during the winter, the likelihood decreases that snow will fall and thus add to any existing snow cover.
-->irrelevant
(D) The nutrients on which algae rely are common in seawater throughout the Antarctic.
-->irrelevant
(E) More sunlight filters through a layer of ice than through an equally thick layer of snow.
-->irrelevant
3
presume:
R employee do not have high c.
(A) early treatment of high cholesterol does not entirely eliminate the possibility of a stroke later in life
-->irrelevant
(B) the mass media regularly feature stories encouraging people to maintain diets that are low in cholesterol
-->irrelevant
(C) RenCo has significantly more employees than Garnet has
-->irrelevant
(D) RenCo's employees are unlikely to have higher cholesterol levels than Garnet's employees
-->maybe correct
(E) the average length of time an employee stays with RenCo is less than it is with Garnet
-->irrelevant
4presume:
branches can not 100% convert into wood chips.
(A) Converting the branches into wood chips at the logging site would require transporting a fully assembled wood-chipping machine to and from the site.
-->weaken
(B) It would be more economical to debranch the trees at the factory where the fiberboard is manufactured.
-->correct.
(C) The debranching of trees and the conversion of the branches into chips are the only stages in the processing of branches that it would be in United's economic advantage to perform at the logging site.
-->weaken
(D) Transportation costs from the logging site to the factory that are determined by volume of cargo would be lower if the conversion into chips is done at the logging site rather than at the factory.
-->weaken
(E) In the wood-processing industry, branches are used only for the production of wood chips for fiberboard.
-->irrelevant
5presume:
Economic stafnation will lead to lower requirment on the oil,which will affect the exports.
(A) the price of oil is expected to remain relatively stable over the next several years
-->support
(B) the improvement in the economic situation in Utrania is expected to result in a dramatic increase in the proportion of Utranians who own automobiles
-->support
(C) most of the investment in new oil fields in Utrania is expected to come from foreign sources
-->correct
(D) new technology is available to recover oil from old oil fields formerly regarded as depleted
-->support
(E) many of the new oil fields in Utrania are likely to be as productive as those that were developed during the period when Utrania was a major oil exporter
-->irrelevant
我要加群 加入组织
作者: 二楼往下掉 时间: 2012-6-16 23:51
(1)47s support
(2)逻辑链
Background:
At most jewelry stores, the person assessing the diamond is the person selling it so you can see why an assessor might say that a diamond is of higher quality than it really is.
Primes:
all diamonds sold at Gem World are certified in writing
Conclusion:
you’re assured of a fair price when purchasing a diamond from Gem World.
(3)有很多钻石的售卖的价格与收购的价格差不多
(4)A:说明G这家店的优势不是优势,大家都有,loser
B:说明G家的这个鉴定很可信,contender
C:文中没有比较G家与其他珠宝店的宝石的质量,而是比较了性价比,loser
D:说明宝石市场价格波动很大,但是没提到文中比较的性价比的问题,loser
E:说明G家的鉴定是由独立机构完成的,所以比较可信,contender
B中算不算权威观点?所以,选E
今儿家里电脑突然坏了。。。我就只能用爱拍的先把精炼做了。。。剩下的明儿修了电脑再发上来吧。。。
话说,磕磕绊绊二十几天中午做完了一季,真心不容易,我就是个没什么耐性的人。不过做完不是目的,这不知不觉也做了一百道题了,需要好好总结下。加油!!!!!
补上剩下四道题
2.
(1)55s explain
(2)逻辑链
Background:
Microscopicplants called algae grow inside the top layer of sea ice in the Antarctic ifenough sunlight reaches that layer of ice and enough seawater, which containsnutrients, reaches that layer after splashing onto the surface.
Fact:
moderatesnow cover reduces the sunlight that filters into the top layer, sea ice withmoderate snow cover typically contains even more algae in the top layer thandoes sea ice with less snow cover.
(3)雪的覆盖会提高温度,导致alga数量多
(4)A:雪的积累导致冰面下沉可以接触更多的海水,contender
B:只讨论南极附近的海水,跟别的地方的没关系,loser
C:文章要讨论的是雪的覆盖与alga的问题,不是雪会覆盖在哪里,loser
D:无关选项,loser
E:如果E对,应该没有雪或者雪少的地方alga比较多,loser
所以,选A
3.
(1)30s explain
(2)逻辑链
Background:
Garnetand RenCo each provide health care for their employees.
Fact1:
Garnetpays for both testing of its employees' cholesterol levels and treatment ofhigh cholesterol. This policy saves Garnet money, since high cholesterol leftuntreated for many years leads to conditions that require very expensivetreatment.
Fact2:
RenCodoes not have the same financial incentive to adopt such a policy.
?
(3)R公司的员工流动性比较大,所以很多员工并未在公司工作很长时间,所以公司不会提供治疗费用。
(4)A:不能完全消除但是也能消除一部分,所以总的来说治疗还是有效果的,loser
B:大量媒体的宣传不能解释两个公司的差异,loser
C:R有更多的员工,那么多年以后患病的人数可能会更多,更应该提早治疗,loser
D:R员工患病的概率低,低也应该提前治疗啊,因为总会有患病的可能性,如果患病就是很大一笔费用,loser
E:同预测,contender
所以,选E
4.
(1)1’05’’ mustbe true
(2)逻辑链
Facts:
<1>United Lumber will use trees from its forests for two products. The treetrunks will be used for lumber and the branches converted into wood chips tomake fiberboard.
<2> The cost of this conversion would be the same whether done at the loggingsite, where the trees are debranched, or at United's factory.
<3> wood chips occupy less than half the volume of the branches from whichthey are made.
(4)A:无关选项,loser
B:没有提到在工厂具体如何制造,loser
C:没有说这是优点,loser
D:因为同样的成本下,chips更省地方,所以运输成本更低,contender
E:没说branches还有没有其他用处,loser
所以,选D
5.
(1)44s explain
(2)逻辑链
Background:
<1>Utrania was formerly a major petroleum exporter, but in recent decadeseconomic stagnation and restrictive regulations inhibited investment in new oilfields.
<2>In consequence, Utranian oil exports dropped steadily as old fields becamedepleted.
<3>Utrania's currently improving economic situation, together withless-restrictive regulations, will undoubtedly result in the rapid developmentof new fields.
Primes:
?
Conclusion:
Itwould be premature to conclude that the rapid development of new fields willresult in higher oil exports
(3)会有新的竞争对手,新的市场规则
(4)A:保持价格稳定不会影响U的复苏速度,loser
B:新的经济环境,汽车很重要,contender
C:与是否来自外国资源没关系,loser
D:新技术表明可以从旧的油田得到油,这必然会导致U的复苏,loser
E:很多新油田都有很好的生产前景,所以有理由相信U会复苏,loser
所以,选B
作者: jetyxo 时间: 2012-6-22 18:38
1 27" background: the man assessing the diamond is the man selling it--->he will assessing it as more good than true
premise: all diamond in GM are certificated in writing
conclusion: they will be sold at fair price
support? the certificate will not unfairly say that the quality of the diamond is more better than they really are
answer:E
--------------------------------------------------------------
2 28" premise: algae grow need 1)sunlight 2) seawater contained nutrition
fact: there was a snow cover the area that reduce the sunlight, but algae in this area grown more than other areas
reason? A sown---> lower location ---> more seawater --->more nutrition
---------------------------------------------------------
3 22" bcakground: G supply health care for its employees ,including test and treatment,beacause long period untreatment will lead to more expensive treatment
fact: M does not do like G ,reason?--->employees in M just work short time
B C D irrelevant ; A ,early treatment will not entirely reduce the possibility ---> no relationship with the cost of treatment answer: E
---------------------------------------------------------
4 29" background: branches---> wood chips---> fiberboard
cost of conversions is the same in factory as in forest
the volum of wood chips is harf of that of branches
infer? ----> D refers the volum
---------------------------------------------------------------
5 34" background: economic stagnation and regulation prohibit from inveting in new oil field--->U export reduce when oil field depleted
fact: economic stimulation and less-restrictive regulation will not lead to increase in export
reason?--->B export--->domestic sale
作者: emmasy 时间: 2012-7-9 22:19
1.
facts:all diamonds sold at Gem World are certified in writing
conclusion:you’re assured of a fair price when purchasing a diamond from Gem World.
a.irrevelant
b.weaken
c.irrevelant
d.weaken
e.support
2.
a.correct
3.
e.correct
4.
d.correct
5.
e.correct
作者: LuckyYolandaLi 时间: 2012-7-12 22:50
1.
1)计时:s
2)逻辑链:
Premise: At most jewelry stores, the person assessing the diamond is the person selling it so
you can see why an assessor might say that a diamond is of higher quality than it really is. All diamonds sold at Gem World are certified in writing
Conclusion: You’re assured of a fair price when purchasing a diamond from Gem World.
3)推测:all diamonds at Gem Would are certificated by official organization.
4)选项分析:选E
A: Irrelevant有点weaken.
B: 。。。
C: Irrelevant.
D: Irrelevant.
E: Correct.
5.
1)计时:53s
2)逻辑链:
Premise: Microscopic plants called algae grow inside the top layer of sea ice in the Antarctic if enough sunlight reaches that layer of ice and enough seawater, which contains nutrients, reaches that layer after splashing onto the surface.
Counter-Premise: Moderate snow cover reduces the sunlight that filters into the top layer.
Conclusion: Sea ice with moderate snow cover typically contains even more algae in the top layer than does sea ice with less snow cover.
3)推测:moderate sunlight, neither much nor little, is helpful to the growth of algae.
4)选项分析:选A
A: Correct.
B: Irrelevant.
C: Irrelevant.
D: Irrelevant.
E: Irrelevant.有点Counteractive.
6.
1)计时:45s
2)逻辑链:
Premise: Garnet and RenCo each provide health care for their employees. Garnet pays for both testing of its employees' cholesterol levels and treatment of high cholesterol. This policy saves Garnet money, since high cholesterol left untreated for many years leads to conditions that require very expensive treatment.
Conclusion: RenCo does not have the same financial incentive to adopt such a policy.
3)推测:无思路。。。
4)选项分析:选A(正确答案:E)
B C D E感觉都无关呢。。。
7.
1)计时:50s
2)逻辑链:
不知道这几句有什么联系。。。
3)推测:。。。
4)选项分析:选D
A: wood-chipping machine——not mentioned.
B: cannot be inferred.
C: the only——too absolute
D: correct
E: only——too absolute
8.
1)计时:58s
2)逻辑链:
Background information: Utrania was formerly a major petroleum exporter, but in recent decades economic stagnation and restrictive regulations inhibited investment in new oil fields.
Premise: Utrania's currently improving economic situation, together with less-restrictive regulations, will undoubtedly result in the rapid development of new fields.
Conclusion: It would be premature to conclude that the rapid development of new fields will result in higher oil exports.
3)推测:
4)选项分析:选B
A: Irrelevant.
B: Correct.
C: Irrelevant.
D: Irrelevant有点Counteractive.
E: Counteractive.
作者: cleotina 时间: 2012-7-15 20:58
46''
support
1 in most of the stores the person assess the diamonds is the person who sell it, so most of the time you will be told the quality is higher than it is
2 In the GW, all the diamonds are certified in writing, you will buy a diamond with equal quality.
Guess:more evidence to prove the others stores will high evaluate the diamonds. so the main point is whether the diamonds are certificated by the interdependent
OA: E
46''
discrepancy
1: algea grow in the first lyer of ice with enough sunlight and enough seawater containing nutrients
2: sea ice with snow cover tend to have more algea, although less eunlight fliter in, than the place with less snow cover
Guess: the place with a thick snow cover tend to contain more seawater for some reason
OA: A
36''
explain the discrepancy
1, G and R both support health care for their employees
2, G provide check and healing for a special kind of desease. since if the disease not found and treated early will spend more money
3, R do not take use the same method .
Guess: the employee in R not tend to have the desease
OA: E
D: very tricky
1'
must be true
1 a tree has two products, one is trunk and another is branch, which is to made wood chips
2 the fee are the same in the logging site and in the factory
3 but the volum of the wood chips will be 50% of the branch
guess: economic is to made the branch into chip as soon as posible, especially before transfering.
OA: D
C: only is too extremed
50'
explain
1 U export oil. since the political stuff it can not obtain new field. as the depleted of the old field, its exported will drop
2 the way to sevle is to dig the new field and thus enlarge the exportation
3 however, it may not a efficient way to solve the problem
Guess: some other factors restraited the amount of the exportation
OA: C
错了,我理解错了选项的意思 资金来自国外并不代表油田一定也在国外, 是我想太多。
作者: Donts 时间: 2012-7-19 09:41
1.计时32''premise:the jewelry is normally assessed by those who sell it at a jewelry store, thus the assessment is usually better than its quality.
conclusion:Since all diamond sold in G are qualified in writing, the price is fair.
推测:把漏洞补好。qualified in writing by other authority agency.
The reasoning in the advertisement would be most strengthened if which one of the following were true?
(A) Many jewelry stores other than Gem World also provide written certification of the quality of their diamonds.无关。
(B) The certifications of diamonds at Gem World are written by people with years of experience in appraising gems.有削弱的意味。
(C) The diamonds sold at Gem World are generally of higher quality than those sold at other jewelry stores.与qualified in writing无关。
(D) The diamond market is so volatile that prices of the most expensive diamonds can change by hundreds of dollars from one day to the next.无关。
(E) The written certifications of diamonds at Gem World are provided by an independent company of gem specialists.符合推测。
2.计时50''
premise:A grows inside the top layer of sea ice with enough sunlight and B;moderate snow will reduce sunlight.
conclusion:in moderate snow top layer of sea ice, the cover contains more A than does sea ice with less snow cover.
推测:雪覆盖越多,B越多,对B的需求更大?
选择A
3.计时37''
G and R both provide health care for their employees. G pays both testing C and high treatment. this policy saves G money because the illness left many years will lead to more expenditure. however, R does not have financial incentive to implement such a policy.
推测:R和G的不同点。
选择E
4.计时49''
the UL use tree for two products: lumber and wood chips. the cost of the conversion is the same whether done in logging or in the factory. however, the chips will occupy less than half volume of the branches from which they are made.
推测:占用更少volume可以盈利,或者说是公司目的。
选择D
5.计时49''
the U is a major petroleum exporter, but recently the economy recession and restrictive regulations inhibit investment in new old fields. consequently, U's old export decreased steadily. U's current improving economy and less-restrictive regulation will definitely lead to rapid development of new fields.however the conclusion is premature because
推测:别的石油进口国状况并不好。
选择C
订正5:B
原因:国内需求量多了,都满足内需去了,export可能就不会太多。要注意点是更多的石油田不一定有更多的出口。
问题:B的错误?
总结:
看到前面是advertisement就要敏感可能有flaw:One quirk of the LSAT is that every stimulus in LSAT history that has been preceded by this word has contained faulty or deceptive logic.
可以尝试把自己放到题目的场景中 If you were standing there in the store and you were told that Gem World has written certification, wouldn’t you ask who does the certification
作者: TICKCOCK 时间: 2012-8-17 01:36
1. E
2. A
3. E
4. D
5. B
完满的结束~
作者: yzrodman 时间: 2012-8-17 10:45
加入加入~
Background:At most jewelry stores, the person assessing the diamond is the person selling it so
you can see why an assessor might say that a diamond is of higher quality than it really is.
Premise:all diamonds sold at Gem World are certified in writing
Conclusion:you’re assured of a fair price when purchasing a diamond from Gem
World.
(A) Many jewelry stores other than Gem World also
provide written certification of the quality of
their diamonds. 看似加强,逻辑链依然有问题,问题在于certification是否真实有效
(B) The certifications of diamonds at Gem World
are written by people with years of
experience in appraising gems. 备选答案之一,在和E比较时发现E更有权威性和说服力
(C) The diamonds sold at Gem World are generally
of higher quality than those sold at other
jewelry stores. 无证据证明
(D) The diamond market is so volatile that prices of
the most expensive diamonds can change by
hundreds of dollars from one day to the next. 更蛋疼的答案~
(E) The written certifications of diamonds at Gem
World are provided by an independent
company of gem specialists. 最佳答案~我选E
作者: wanggang0411 时间: 2012-8-28 15:32
27
Premise: The diamond sell in a higher value by a people who sell it.
Premise: The diamond is certified in wirting.
Conclusion: The diamond is in a fair price.
Inference:
The author commits a gratuitous assumption that the certification assures a fair price.
However, no evidence is stated in the argumentto support this assumption.
In fact, this is not necessarily the case. For example, it is more likely that the certification is written by the person who sells the diamond,
and then no fair price will be guaranteed. Therefore, this argument is unwarranted without ruling out such possibility.
To strengthen this argument, author could provide a evidence that the certification is released from a authority which has no interest relations.
37
Premise: Algae rests on light to grow.
Premise: Algae grows faster in the place where moderate snow cover the algae than in the place without snow.
Inference: A
A
40
Premise: Garnet pays for two testing.
Premise: Renco does not have the same finaicial incentives.
Inference:E
E
1:08
I spent long time and repeat many times on this sentence: wood chips occupy less than half of the volume of the branches from which they are made.
Premise: The cost of convertion would be the same at the logging site, or at the factory.
Premise: the cost of the branches are less at the logging site than at the factory.
Inference:E
D: I misinterpretated the meaning of the sentence that wood chips occupy less than half of the volume of the branches from which they are made.
37
Premise: Because of the depletion of oil field and restrictive regulation, oil exports dropped steadily.
Premise: Loosing the regulation and developing the new field.
Conclusion: those measures may not result in higher oil exports.
Inference: B this is the only reason that impact the exports of oil.
B
作者: 我心匪席 时间: 2012-9-4 16:11
Time: 45”
Background Information: Algae grow inside the top layer of sea ice with enough sunlight and nutrient sea water
Premise: Snow cover the top layer
Conclusion: More algae in the top layer
Prephase: Snow cover has same nutrient which sunlight can offer.
A-----right support
B-----irrelevant
C-----irrelevant
D-----irrelevant
E-----irrelevant
=============================================
Time: 21”
Background Information: G and R provide health for employee. G pays for both testing and treatment for high cholesterol. G saves money for high cholesterol left untreated will require more money.
Premise: ?
Conclusion: R has the no such financial incentive to adopt such a policy.
Prephase: Employees work in R in a short time.
A-----irrelevant
B-----irrelevant
C-----irrelevant
D-----irrelevant
E-----right
===============================================
Time: 27”
Background Information: Trees can be used for lumber and branches converted into wood chips. The cost of conversion would be same whether done in logging site or at factory.
Premise: Wood chips occupy less space than branches.
Conclusion: ?
Prephase: Transporting wood chips will cost less.
A-----irrelevant
B-----irrelevant
C-----irrelevant
D-----right
E-----irrelevant
==============================================
Time: 45”
Background Information: Utrania was a petroleum exporter; economic stagnation and regulations inhibit investment in new oil fields. Old fields became depleted, therefore oil experts dropped. U’s improving economic situation and less restrictive regulations will result in development of new fields.
Premise: ?
Conclusion: It would be premature to conclude that the rapid development of new fields will result in higher oil exports.
Prephase: Needs for oil at home are increasing.
A-----irrelevant
B-----right
C-----irrelevant
D-----irrelevant
E-----irrelevant
作者: Cassiezhao 时间: 2012-10-15 18:42
我做了超过20天了都,可是没粘上来楼主
作者: ElenW 时间: 2012-10-21 09:36
精练(32.1s)
bg:the person assessing the diamond is the person selling it
pre:all diamonds sold at Gem World are certified in writing
con:be assured of a fair price when purchasing a diamond from Gem
World.
(strengthen)
PRE:The certification is not from the same company.(解析中说The certification may have no impact on the
actual price of the diamond, or perhaps it could even be used to raise the price
unjustly.这是我忽略掉的一点)
Ans:E
A--no impact
B--no impact
C--not the issue
D--no impact
逻辑链5(32.2s)
再看了一遍题目
Bg:Microscopic plants called algae grow inside the top layer of sea ice in the Antarctic if enough sunlight reaches that layer of ice and enough seawater
pre: Even though moderate snow cover reduces the sunlight that filters into the top layer
con:sea ice with moderate snow cover typically contains even more algae in the top layer than does sea ice with less snow cover.
(strengthen)
Ans:B--correct A
这个题想问问大家,读了好久遍都没有懂~~~
6(29.5's)
Bg:Garnet and RenCo each provide health care for their employees
pre1:high cholesterol left untreated for many years leads to conditions that require very expensive treatment
con:This policy saves Garnet money
pre2:未知
con:R does not have the same financial incentive to adopt such a policy
Ans:E
A--not the issue
B--irrelevant
C--irrelevant
D--irrelevant
作者: srafcatt 时间: 2012-10-21 14:37
精炼
1. strenghten 30s
jewlery store tend to exaggerate the diamonds' value.
while in gem world you are assured of the fair price
my attempt:the writing is valued by all the gem sellers.
choice:E
A weaken
B irrelevant
C irrelevant
D weaken
E correct
---------------------------------------------------------
逻辑链
1.explaination 23s
premise:algea need to live with light and nutrients reachable ice layer.
conclusion:ice layer with less light because of moderate snow will increase the number of algea.
my attempt:the snow melts it will contain more nutrients in it.
choice:A
A correct
B irrelevant
C irrelevant
D irrelevant
E weaken
2.fill 35s
garnet benefits from the policy,but renco cannot copy with the same finacial incentives
because of the economy of scale
answer
A specific sample,so not convincing
B irrelevant
C so what
D correct
E so what
3.strenghten 25s
cost of making chips is the same no matter where it is,but the chips will occupy less space.
so my conclusion is: make the chips right there where the branches are found.
answer
A irrelevant
B incorrect
C incorrect
D correct
E irrelevant
4.fill 41s
premise: investment in new oil fields is limited.so exports dropped
recently the economic and the regulations are all good
conclusion:exports will not be developped
answer:B
A irrelevant
B correct
C strenghten
D strenghten
E strenghten
作者: srafcatt 时间: 2012-10-21 14:42
6. (24353-!-item-!-188;#058&001207)
Which of the following most logically completes the passage?
Garnet and RenCo each provide health care for their employees. Garnet pays for both testing of its employees' cholesterol levels and treatment of high cholesterol. This policy saves Garnet money, since high cholesterol left untreated for many years leads to conditions that require very expensive treatment. However, RenCo does not have the same financial incentive to adopt such a policy, because __________.
(A) early treatment of high cholesterol does not entirely eliminate the possibility of a stroke later in life
(B) the mass media regularly feature stories encouraging people to maintain diets that are low in cholesterol
(C) RenCo has significantly more employees than Garnet has
(D) RenCo's employees are unlikely to have higher cholesterol levels than Garnet's employees
(E) the average length of time an employee stays with RenCo is less than it is with Garnet
这是一题解释矛盾题。两个公司的做法有矛盾,G公司的做法给出了理由,但是题目要求我们去解释R公司的做法合理性(主要强调的是R公司有没有financial incentive)。错误选项D,只是再说R公司的员工的C水平不会高于G公司的员工,即小于等于;并没有充分的理由说明为什么R公司不学G公司的做法是由financial incentive的。正确选项应为E,改选项说明了R公司不需要这耗费额外费用的检查,能够省钱,因为G公司员工长期工作后的医疗费用节省的理由在R公司不成立,因为R公司不需要老员工。
作者: srafcatt 时间: 2012-10-21 14:43
哎呀,终于有一次只错一道题了。
作者: oytt1111 时间: 2012-10-22 17:03
1-20
Time: 35s
Premise: the person assessing the diamond is the person selling it, so diamond quality might be exaggerated.
Premise: diamonds sold at GW are certified in writing
Conclusion: You are assured of a fair price when purchasing in GW
Prephrase: Lack of some premises which bridge the premise and conclusion
Answer: E
A. irrelevant
B. people with years of experience in appraising gem might be GW sales themselves.
C. higher quality will not assure fair price and might also be exaggerated.
D.irrelevant
E. right answer.
5
Time: 52s
Premise: Algae grow in side the top layer of sea ice if sunlight and seawater reach that layer of ice
Premise: Moderrate snow cover reduces the sunlight that filters into the top layer
Conclusion: Sea ice with moderate snow cover typically contains more algae than with less snow cover
Premise: Lack of premise to bridge
Answer: E
6
Time: 50s
Premise: G pays for both test and treatment..
Premise: This policy saves G money because long untreated leads to expensive treatment.
Conclusion: R does not have same policy
Answer: E
7
Time: 42s
Premise: Two products are made. Tree trunks for lumber and branches for wood chips.
Premise: Cost are the same whether convert branches into chips at logging site or at factory.
Premise: chips occupy less than branches
Answer: D
8
Time: 63s
Premise: U exports petroleum but in recent decades economic stagnation and regulations inhibited investment in new oil fields.
Premise: U oil exports dropped steadily as old fields became depleted.
Premise: new fields will be developed
Conclusion: it would be premature to conclude that rapid development of new fields will result in higher oil exports.
Answer: B
作者: zxppx 时间: 2013-1-17 09:56
1.
Advertisement: At most jewelry stores, the person
assessing the diamond is the person selling it so
you can see why an assessor might say that a
diamond is of higher quality than it really is.
But because all diamonds sold at Gem World
are certified in writing, you’re assured of a fair
price when purchasing a diamond from Gem
World.
There is a contradiction in the argument, a contradiction that at most jewelry stores, the person assessing the diamond is the person selling it so that the price may be unfair, while Gem World declares that the prices of its diamonds are fair. How to explain it? If the price in Gem World is not assessed or selling by the same person, then it would be much reasonable to understand the declaration of Gem World.
The reasoning in the advertisement would be most
strengthened if which one of the following were true?
(A) Many jewelry stores other than Gem World also
provide written certification of the quality of
their diamonds.
Actually, this answer choice cannot strengthen the argument, since the prices of diamonds in Gem World may be the same unfair as those of other jewelry stores that provide written certification of the quality of their diamonds.
(B) The certifications of diamonds at Gem World
are written by people with years of
experience in appraising gems.
This answer choice does not make sure that these assessing people are not the people who sell diamonds. Perhaps, these persons are hired by Gem World, thus making the argument much less valid.
(C) The diamonds sold at Gem World are generally
of higher quality than those sold at other
jewelry stores.
The quality of diamonds has no effect on the argument.
(D) The diamond market is so volatile that prices of
the most expensive diamonds can change by
hundreds of dollars from one day to the next.
In fact, this answer choice may weaken the argument, since the volatile prices of the diamonds. Besides, the prices of the most expensive diamonds narrow down the scope of the topic, thus making the argument much less valid.
(E) The written certifications of diamonds at Gem
World are provided by an independent
company of gem specialists.
In this case, we can find that these certifications of diamonds are provided by an independent company of gem specialists, different from the information provided in the first sentence. BA
As mentioned above, this answer addresses the separation of the certification writer from the seller and thereby strengthens the reasoning.
5. (24305-!-item-!-188;#058&001204)
Microscopic plants called algae grow inside the top layer of sea ice in the Antarctic if enough sunlight reaches that layer of ice and enough seawater, which contains nutrients, reaches that layer after splashing onto the surface. Even though moderate snow cover reduces the sunlight that filters into the top layer, sea ice with moderate snow cover typically contains even more algae in the top layer than does sea ice with less snow cover.
Which of the following, if true, most helps to explain the apparent discrepancy?
(A) As the weight of accumulated snow forces ice lower in the water, more seawater splashes onto the surface and seeps through the snow.
Yes. In this case, it points out an advantage of the snow, so compared with less snow cover, moderate snow cover is better for algae to grow. BA
(B) Seawater in the Antarctic often contains higher concentrations of nutrients than does seawater in more temperate regions.
Temperate regions are outside the scope of the argument.
(C) As the air temperature around sea ice decreases during the winter, the likelihood decreases that snow will fall and thus add to any existing snow cover.
Actually, this answer choice makes the argument more contradictory, since there will be more snow cover during the winter.
(D) The nutrients on which algae rely are common in seawater throughout the Antarctic.
In fact, this answer choice has no effect on the argument.
(E) More sunlight filters through a layer of ice than through an equally thick layer of snow.
The comparison between a layer of ice and a layer of snow is irrelevant, since the topic is the comparison between a moderate snow cover and a less snow cover.
6. (24353-!-item-!-188;#058&001207)
Which of the following most logically completes the passage?
Garnet and RenCo each provide health care for their employees. Garnet pays for both testing of its employees' cholesterol levels and treatment of high cholesterol. This policy saves Garnet money, since high cholesterol left untreated for many years leads to conditions that require very expensive treatment. However, RenCo does not have the same financial incentive to adopt such a policy, because __________.
(A) early treatment of high cholesterol does not entirely eliminate the possibility of a stroke later in life
Actually, this answer choice seems to weaken the action of Garnet. However, the treatment for stroke may be very expensive for the patients and the company, considering the huge costs and the possibility that health care may eliminate the possibility of a stroke in some degree, so companies still need to provide for health care.
(B) the mass media regularly feature stories encouraging people to maintain diets that are low in cholesterol
In fact, this answer choice is irrelevant.
(C) RenCo has significantly more employees than Garnet has
So RenCo need do more with health care than Garnet need.
(D) RenCo's employees are unlikely to have higher cholesterol levels than Garnet's employees
At first glance, it seems to be a contender, but a more thorough analysis reveals that the evidence is misleading. Perhaps, RenCo’s employees have the same cholesterol levels as Garnet’s employees do, thus making the argument much less valid.
(E) the average length of time an employee stays with RenCo is less than it is with Garnet
Yes. In this case, since a stroke is more likely to happen in a person’s later life, RenCo has less incentive to provide health care just as Garnet do. BA
7. (24401-!-item-!-188;#058&001291)
United Lumber will use trees from its forests for two products. The tree trunks will be used for lumber and the branches converted into wood chips to make fiberboard. The cost of this conversion would be the same whether done at the logging site, where the trees are debranched, or at United's factory. However, wood chips occupy less than half the volume of the branches from which they are made.
The information given, if accurate, most strongly supports which of the following?
(A) Converting the branches into wood chips at the logging site would require transporting a fully assembled wood-chipping machine to and from the site.
Nothing about a fully assembled wood-chipping machine has been provided in the argument.
(B) It would be more economical to debranch the trees at the factory where the fiberboard is manufactured.
Actually, it is the opposite answer, since wood chips occupy less than half the volume of the branches from which they are made. If wood chips are made at the logging site and the cost of this conversion would be the same whether done at the logging site or at United’s factory, more transportation fees can be reduced.
(C) The debranching of trees and the conversion of the branches into chips are the only stages in the processing of branches that it would be in United's economic advantage to perform at the logging site.
This is not relevant.
(D) Transportation costs from the logging site to the factory that are determined by volume of cargo would be lower if the conversion into chips is done at the logging site rather than at the factory.
Yes, since much volume of cargo can be reduced if the conversion into chips is done at the logging site rather than at the factory, the transportation costs will be reduced, thus making the argument much reasonable. BA
(E) In the wood-processing industry, branches are used only for the production of wood chips for fiberboard.
Shell Game: the argument talks about United Lumber, rather than the wood-processing industry.
8. (24449-!-item-!-188;#058&001338)
Which of the following most logically completes the argument?
Utrania was formerly a major petroleum exporter, but in recent decades economic stagnation and restrictive regulations inhibited investment in new oil fields. In consequence, Utranian oil exports dropped steadily as old fields became depleted. Utrania's currently improving economic situation, together with less-restrictive regulations, will undoubtedly result in the rapid development of new fields. However, it would be premature to conclude that the rapid development of new fields will result in higher oil exports, because __________.
(A) the price of oil is expected to remain relatively stable over the next several years
Actually, this answer choice strengthens the argument, rather than weakens.
(B) the improvement in the economic situation in Utrania is expected to result in a dramatic increase in the proportion of Utranians who own automobiles
The increase in the proportion of Utranians who own automobiles would depress the oil export, even with the rapid development of new fields. This way to weaken is to introduce another factor, which would influence the result. BA
(C) most of the investment in new oil fields in Utrania is expected to come from foreign sources
Actually, this answer choice has no effect on the argument.
(D) new technology is available to recover oil from old oil fields formerly regarded as depleted
It seems to strengthen the argument. Besides, it is irrelevant to the topic of the development of new fields.
(E) many of the new oil fields in Utrania are likely to be as productive as those that were developed during the period when Utrania was a major oil exporter
In fact, this answer choice seems to strengthen the argument, rather than weaken.
作者: pennyz 时间: 2013-2-27 22:23
1:29s strengthen
E
background: daimond assessment is not reliable,if it is said
premise:the gem world is written
conclusion:the diamond in gem world is reliable
rephrase: the written is different from said in nature
(A) Many jewelry stores other than Gem World also
provide written certification of the quality of
their diamonds.-----weaken
(B) The certifications of diamonds at Gem World
are written by people with years of
experience in appraising gems.-----kind of support
(C) The diamonds sold at Gem World are generally
of higher quality than those sold at other
jewelry stores.-------irrelevant
(D) The diamond market is so volatile that prices of
the most expensive diamonds can change by
hundreds of dollars from one day to the next.-------irrelevant
(E) The written certifications of diamonds at Gem
World are provided by an independent
company of gem specialists.--------correct
作者: pennyz 时间: 2013-3-1 17:38
2:38s
explain
e---deny the premise
a
background:with snow has algae/algae more---if enough sunlight and enough water
premise:more snow less sunlight
phenomenon:more algea in snow than less snow
rephrase: too much light in less snow
think about seawater
A) As the weight of accumulated snow forces ice lower in the water, more seawater splashes onto the surface and seeps through the snow.-----correct
(B) Seawater in the Antarctic often contains higher concentrations of nutrients than does seawater in more temperate regions.-------irrelevant
(C) As the air temperature around sea ice decreases during the winter, the likelihood decreases that snow will fall and thus add to any existing snow cover.----irrelevant
(D) The nutrients on which algae rely are common in seawater throughout the Antarctic.--irrelevant
(E) More sunlight filters through a layer of ice than through an equally thick layer of snow.
---contradict with the premise
1:30s conclusion's premise
e
background:g/r both provide medical care
g provide c,because c not treated will cost more later
premise:?
conclusion:r won't give c to employer
rephrase:employer of c will not stay that long
?
3:42s
conclusion
d
premise:two :lumber wc
wc cost same on logging place and factory
but wc occupy less space
conclusion:?
better retain wc than lumber
(A) Converting the branches into wood chips at the logging site would require transporting a fully assembled wood-chipping machine to and from the site.-------
(B) It would be more economical to debranch the trees at the factory where the fiberboard is manufactured.--------
(C) The debranching of trees and the conversion of the branches into chips are the only stages in the processing of branches that it would be in United's economic advantage to perform at the logging site.
(D) Transportation costs from the logging site to the factory that are determined by volume of cargo would be lower if the conversion into chips is done at the logging site rather than at the factory.
(E) In the wood-processing industry, branches are used only for the production of wood chips for fiberboard.
2:15s
e
background: stagnant environment& restriction prohibite the new oil field
old field depleted---->less exportion
premise: better economic situation and remove of restriction
conclusion:can not cause more export
rephrase: new field may not have so much oil as the old one
the need for oil outboard may not increase
作者: kid381 时间: 2013-7-15 11:43
7/15/13
1-20
196 26’’
At most place, diamond are certified by the person selling you the necklace, so you do not expect fair evaluation, but one jewelry store, X, gives you a paper-based certify, so you can trust X.
Strengthen: X will be accused of defraud if they evaluate the jewelry unfairly (obviously E).
197 45’’
One kind of algae lives in the ice sheets floated on the surface of the sea, two requirements for its living: sunshine and nutrients. Though the moderate snow reduces the sunshine, the algae still lives well. Why?
Explain why: moderate snow make the algae obtain more seawater, which contains nutrients (A)
198 21’’
G and R both provide health care for the employees. Garnet pays for tests of cholesterol and treatment of cholesterol. R does not have the incentive. Why?
Employee of R typically leaves R after working for several years (E).
199 45’’
UL cut trees for two products: trunk for lumber and branches into wood chips to make fiberboard. The conversion cost the same in different place. However, more than half of chips are not made in the cutting place.
Conclusion: the transportation costs of trunks are higher than the cost of branches (D).
200 45’’
Utrania is a big oil exporter, these years since stricter regulation, the new oil excavation is limited and the old oil well depletes, so the export of Utrania declines. But even though Utrania can excavate new oil well, it is premature to say that Ultrania will have more oil exportation.
Explain why: the cost of oil excavation enhances, so Ultrania cannot export as much as oil as they did in the past (B. B explains that the internal demand is increasing).
呼呼,终于练完了,上考场去吧~
作者: Elisha728 时间: 2013-9-3 07:06
8'54''
EAEDB
ALL CORRECT
作者: lyrsilvia 时间: 2013-9-21 20:43
E AEDB
1. 26'
BG: the person assessing the diamond is the person selling it so you can see why an assessor might say that a diamond is of higher quality than it really is.
P: all diamonds sold at Gem World are certified in writing
C: you’re assured of a fair price when purchasing a diamond from Gem World.
MUST BE TRUE:
答案:E
(A) Many jewelry stores other than Gem World also provide written certification of the quality of their diamonds.--------what other jewelry stores do does not matter. irrelevant
(B) The certifications of diamonds at Gem World are written by people with years of experience in appraising gems.----------------seems correct. but not as convincing as E.
(C) The diamonds sold at Gem World are generally of higher quality than those sold at other jewelry stores.-------------the quality is not the issue
(D) The diamond market is so volatile that prices of the most expensive diamonds can change by hundreds of dollars from one day to the next.-----------------how the price change does noting to do with consumers' judgement about the fair price. even weaken the conclusion since the price is volatile
(E) The written certifications of diamonds at Gem World are provided by an independent company of gem specialists.---------------CORRECT. if not independent company, the price specialists gave might reflect the company's interest. which means specialists will overvalue the diamond.
2. 32'
P: algae grow inside the top layer of sea ice in the Antarctic if enough sunlight reaches that layer of ice and enough seawater, which contains nutrients, reaches that layer after splashing onto the surface. moderate snow cover reduces the sunlight that filters into the top layer
C: sea ice with moderate snow cover typically contains even more algae in the top layer than does sea ice with less snow cover
EXPLAIN: snow reduce sunlight but brings some advantage for algae, maybe nutrients
答案:A
(A) As the weight of accumulated snow forces ice lower in the water, more seawater splashes onto the surface and seeps through the snow.-----------CORRECT. more seawater=more nutrients
(B) Seawater in the Antarctic often contains higher concentrations of nutrients than does seawater in more temperate regions.------------the comparison between seawater in different regions is irrelevant
(C) As the air temperature around sea ice decreases during the winter, the likelihood decreases that snow will fall and thus add to any existing snow cover.-----------what temperature change leads to is irrelevant
(D) The nutrients on which algae rely are common in seawater throughout the Antarctic.-------------what the nutrients are does not matter at all
(E) More sunlight filters through a layer of ice than through an equally thick layer of snow.-----------------means more snow is a disadvantage, cannot explain the discrepancy.
3. 22'
BG: Garnet and RenCo each provide health care for their employees.
G: pays for both testing of its employees' cholesterol levels and treatment of high cholesterol. This policy saves Garnet money, since high cholesterol left untreated for many years leads to conditions that require very expensive treatment
R: does not have the same financial incentive to adopt such a policy
EXPLAIN: the difference between employers in G and R
答案:E
(A) early treatment of high cholesterol does not entirely eliminate the possibility of a stroke later in life-------------whether entirely eliminate, still has its advantage for saving money. this does not illustrate the difference between G and R
(B) the mass media regularly feature stories encouraging people to maintain diets that are low in cholesterol----------------what mass media does is irrelevant
(C) RenCo has significantly more employees than Garnet has--------------what information can be drawn from "more employees"? this statement just slightly suggest the financial incentive to adopt such a policy in R
(D) RenCo's employees are unlikely to have higher cholesterol levels than Garnet's employees----------seems logical, but does higher cholesterol levels mean less patients? not exactly correct. higher≠less
(E) the average length of time an employee stays with RenCo is less than it is with Garnet----------------------CORRECT. short career life means less chance to have employees that develop cholesterol.
4. 30'
P: tree trunks will be used for lumber and the branches converted into wood chips to make fiberboard
cost of this conversion would be the same whether done at the logging site, or at United's factory
wood chips occupy less than half the volume of the branches from which they are made.
SUPPORT: chips are more economical for transporting ( ALWAYS BE CAREFUL ABOUT THE "HOWEVER")
答案:D
(A) Converting the branches into wood chips at the logging site would require transporting a fully assembled wood-chipping machine to and from the site.-------------the transporting of machine is not discussed in the question even though it seems to increase the cost
(B) It would be more economical to debranch the trees at the factory where the fiberboard is manufactured.-------------this statement just violate the HOWEVER sentence.
(C) The debranching of trees and the conversion of the branches into chips are the only stages in the processing of branches that it would be in United's economic advantage to perform at the logging site.--------------whether this conversion is the only economic advantage stages is not discussed in the question. WATCH OUT THE IMPORTANT MEANING "ONLY" IN THIS SENTENCE
(D) Transportation costs from the logging site to the factory that are determined by volume of cargo would be lower if the conversion into chips is done at the logging site rather than at the factory.--------------------CORRECT. quite suit the HOWEVER sentence
(E) In the wood-processing industry, branches are used only for the production of wood chips for fiberboard.---------------what the branches are used is irrelevant.
5. 33'
BG: economic stagnation and restrictive regulations inhibited investment in new oil fields. In consequence, Utranian oil exports dropped steadily as old fields became depleted.
P: Utrania's currently improving economic situation, together with less-restrictive regulations, will undoubtedly result in the rapid development of new fields.
C: it would be premature to conclude that the rapid development of new fields will result in higher oil exports
EXPLAIN: new fields≠more export, maybe domestic consume rises
答案:B
(A) the price of oil is expected to remain relatively stable over the next several years---------------what the price is does not affect the export or import. this statement even slightly support the increased export
(B) the improvement in the economic situation in Utrania is expected to result in a dramatic increase in the proportion of Utranians who own automobiles---------------CORRECT. more automobiles mean more domestic consume, may not generate high export
(C) most of the investment in new oil fields in Utrania is expected to come from foreign sources-------------------where the investment comes from does not matter. and this statement even support the more export to foreign sources
(D) new technology is available to recover oil from old oil fields formerly regarded as depleted-----------means more production, support the increasing export
(E) many of the new oil fields in Utrania are likely to be as productive as those that were developed during the period when Utrania was a major oil exporter--------------means more production. support the increasing export.
作者: 览物之情 时间: 2013-10-21 17:11
10月21
1 29s
Advertisement: At most jewelry stores, the person
assessing the diamond is the person selling it so
you can see why an assessor might say that a
diamond is of higher quality than it really is.
premise: But because all diamonds sold at Gem World are certified in writing.
Conclusion:you’re assured of a fair price when purchasing a diamond from Gem
World. P 写鉴定的就不是卖的人吗?
The reasoning in the advertisement would be most
strengthened if which one of the following were true?
(A) Many jewelry stores other than Gem World also
provide written certification of the quality of
their diamonds.-----NM
(B) The certifications of diamonds at Gem World
are written by people with years of
experience in appraising gems.——NM
(C) The diamonds sold at Gem World are generally
of higher quality than those sold at other
jewelry stores.-----NM
(D) The diamond market is so volatile that prices of
the most expensive diamonds can change by
hundreds of dollars from one day to the next.----NM
(E) The written certifications of diamonds at Gem
World are provided by an independent
company of gem specialists.-----Correct
2 39s
Background:Microscopic plants called algae grow inside the top layer of sea ice in the Antarctic if enough sunlight reaches that layer of ice and enough seawater, which contains nutrients, reaches that layer after splashing onto the surface.
Even though moderate snow cover reduces the sunlight that filters into the top layer
现象:sea ice with moderate snow cover typically contains even more algae in the top layer than does sea ice with less snow cover.
Which of the following, if true, most helps to explain the apparent discrepancy?
More nutrients?
(A) As the weight of accumulated snow forces ice lower in the water, more seawater splashes onto the surface and seeps through the snow.
(B) Seawater in the Antarctic often contains higher concentrations of nutrients than does seawater in more temperate regions.----NM
(C) As the air temperature around sea ice decreases during the winter, the likelihood decreases that snow will fall and thus add to any existing snow cover.---NM
(D) The nutrients on which algae rely are common in seawater throughout the Antarctic.----NM
(E) More sunlight filters through a layer of ice than through an equally thick layer of snow.----correct
3 30s
Which of the following most logically completes the passage?
Garnet and RenCo each provide health care for their employees. Garnet pays for both testing of its employees' cholesterol levels and treatment of high cholesterol.
Conclusion:This policy saves Garnet money
Because: high cholesterol left untreated for many years leads to conditions that require very expensive treatment.
However, RenCo does not have the same financial incentive to adopt such a policy, because __________.P 他们的turnover很高
因果推理突破点在原因的特点
(A) early treatment of high cholesterol does not entirely eliminate the possibility of a stroke later in life---NM
(B) the mass media regularly feature stories encouraging people to maintain diets that are low in cholesterol---NM
(C) RenCo has significantly more employees than Garnet has---NM
(D) RenCo's employees are unlikely to have higher cholesterol levels than Garnet's employees----NM
(E) the average length of time an employee stays with RenCo is less than it is with Garnet-----correct
4 42s
Premise: The cost of this conversion would be the same whether done at the logging site, where the trees are debranched, or at United's factory.
Outcome: However, wood chips occupy less than half the volume of the branches from which they are made.
现象解释:为什么他们不再工厂里弄。运输费用
The information given, if accurate, most strongly supports which of the following?
(A) Converting the branches into wood chips at the logging site would require transporting a fully assembled wood-chipping machine to and from the site.--wrong
(B) It would be more economical to debranch the trees at the factory where the fiberboard is manufactured.--wrong
(C) The debranching of trees and the conversion of the branches into chips are the only stages in the processing of branches that it would be in United's economic advantage to perform at the logging site.----susepct
(D) Transportation costs from the logging site to the factory that are determined by volume of cargo would be lower if the conversion into chips is done at the logging site rather than at the factory.-----correct
(E) In the wood-processing industry, branches are used only for the production of wood chips for fiberboard.---NM
5 31s
hich of the following most logically completes the argument?
premise:Utrania's currently improving economic situation, together with less-restrictive regulations, will undoubtedly result in the rapid development of new fields.
Conclusion: However, it would be premature to conclude that the rapid development of new fields will result in higher oil exports, because __内需增加
(A) the price of oil is expected to remain relatively stable over the next several years
(B) the improvement in the economic situation in Utrania is expected to result in a dramatic increase in the proportion of Utranians who own automobiles----correct
(C) most of the investment in new oil fields in Utrania is expected to come from foreign sources----NM
(D) new technology is available to recover oil from old oil fields formerly regarded as depleted----NM
(E) many of the new oil fields in Utrania are likely to be as productive as those that were developed during the period when Utrania was a major oil exporter—NM
作者: w.melhere 时间: 2013-11-2 13:43
1.32 1.03
2. 2'07
3.33‘
4. 57' 1.37
5.2.24' 做题花了这么久,而且还错了...不过这道题错的真是心服口服啊,做题的时候看完B完全没找到感觉,后来再仔细读的时候才想到。
B选项说,U的经济快速发展,拥有汽车的人的比例会很大。也就是说国内对石油的消耗会大幅增长。故用于出口的石油就不一定能增长了。
btw,pace到底怎么掌握好啊? 每道题在规定的时间内就可以了么? 觉得还是没有很稳定的pace
作者: yuehuasunday 时间: 2013-11-21 16:51
1-20
1.
Background: Advertisement: At most jewelry stores, the person
assessing the diamond is the person selling it
Premise: so
you can see why an assessor might say that a
diamond is of higher quality than it really is.
Conclusion: But because all diamonds sold at Gem World
are certified in writing, you’re assured of a fair
price when purchasing a diamond from Gem
World.
(E) Correct, the best choice
2.
Background: Microscopic plants called algae grow inside the top layer of sea ice in the Antarctic if enough sunlight reaches that layer of ice and enough seawater
Premise: Even though moderate snow cover reduces the sunlight that filters into the top layer
Conclusion: sea ice with moderate snow cover typically contains even more algae in the top layer than does sea ice with less snow cover.
(A) Correct
3.
Background: Garnet and RenCo each provide health care for their employees.
Premise: Garnet pays for both testing of its employees' cholesterol levels and treatment of high cholesterol.
Conclusion: This policy saves Garnet money, since high cholesterol left untreated for many years leads to conditions that require very expensive treatment.
(E) Correct
4.
Background: United Lumber will use trees from its forests for two products.
Premise: The tree trunks will be used for lumber and the branches converted into wood chips to make fiberboard.
Conclusion: The cost of this conversion would be the same whether done at the logging site, where the trees are debranched, or at United's factory.
(D) Correct
5.
Background: Utrania was formerly a major petroleum exporter, but in recent decades economic stagnation and restrictive regulations inhibited investment in new oil fields.
Premise: In consequence, Utranian oil exports dropped steadily as old fields became depleted.
Conclusion: Utrania's currently improving economic situation, together with less-restrictive regulations, will undoubtedly result in the rapid development of new fields. However, it would be premature to conclude that the rapid development of new fields will result in higher oil exports, because __________.
(B) Correct
作者: irenetopia 时间: 2013-12-3 13:58
1. 1:30
Advertisement: At most jewelry stores, the person
assessing the diamond is the person selling it so
you can see why an assessor might say that a
diamond is of higher quality than it really is.
But because all diamonds sold at Gem World
are certified in writing, you’re assured of a fair
price when purchasing a diamond from Gem
World.
The reasoning in the advertisement would be most
strengthened if which one of the following were true?
逻辑链:大多数珠宝店,一个人评价珠宝总是说比其他人质量好,因为大部分都是自己个人出售的。
但是在GW珠宝店你就不用担心,因为我们有个检测证书。
问加强
思路:检测证书突出了GW的公平性,那么是不是这个检测证书就一定客观?加强客观性的都是support!
(A) Many jewelry stores other than Gem World also
provide written certification of the quality of
their diamonds.——weaken
(B) The certifications of diamonds at Gem World
are written by people with years of
experience in appraising gems.——irrelevant,有点道理,但是不是特别有说服力,留着
(C) The diamonds sold at Gem World are generally
of higher quality than those sold at other
jewelry stores.——觉得重复前提,排除
(D) The diamond market is so volatile that prices of
the most expensive diamonds can change by
hundreds of dollars from one day to the next.——贵跟质量可信赖有什么关系
(E) The written certifications of diamonds at Gem
World are provided by an independent
company of gem specialists.——bingo!因为检测公司是第三方独立,客观加强!
PREP CR 2 逻辑练习
5. 1:45
Microscopic plants called algae grow inside the top layer of sea ice in the Antarctic if enough sunlight reaches that layer of ice and enough seawater, which contains nutrients, reaches that layer after splashing onto the surface. Even though moderate snow cover reduces the sunlight that filters into the top layer, sea ice with moderate snow cover typically contains even more algae in the top layer than does sea ice with less snow cover.
Which of the following, if true, most helps to explain the apparent discrepancy?
逻辑链A植物在top layer of sea ice 上生长,需要两个条件,足够的阳光,足够的海水
但是,即使下雪了,阻挡了阳光的穿透力,但是A植物反而多了
求解释:阳光少了,但是营养更多,说明什么?海水多了!
(A) As the weight of accumulated snow forces ice lower in the water, more seawater splashes onto the surface and seeps through the snow.——bingo!
(B) Seawater in the Antarctic often contains higher concentrations of nutrients than does seawater in more temperate regions.——没有这个和其他常温地区的比较
(C) As the air temperature around sea ice decreases during the winter, the likelihood decreases that snow will fall and thus add to any existing snow cover.——题目是讨论下雪后A多了的原因,不是说雪下不下的原因
(D) The nutrients on which algae rely are common in seawater throughout the Antarctic.——无关
(E) More sunlight filters through a layer of ice than through an equally thick layer of snow.——weaken!
6. 1:45
Which of the following most logically completes the passage?
Garnet and RenCo each provide health care for their employees. Garnet pays for both testing of its employees' cholesterol levels and treatment of high cholesterol. This policy saves Garnet money, since high cholesterol left untreated for many years leads to conditions that require very expensive treatment. However, RenCo does not have the same financial incentive to adopt such a policy, because __________.
逻辑链:G公司和R公司都在给员工进行医保
G测试员工胆固醇并且治疗高胆固醇员工,但这样会让他省钱(如果不治疗多年以后发现花公司更多钱)
但是R不用做这个,因为?
思路:花钱是为了多年以后省钱。那么不花钱是因为什么?没有多年以后这个说法~也就是员工不会呆很久
(A) early treatment of high cholesterol does not entirely eliminate the possibility of a stroke later in life——但是只要早期治疗就能省钱
(B) the mass media regularly feature stories encouraging people to maintain diets that are low in cholesterol——跟做体检有关么
(C) RenCo has significantly more employees than Garnet has——这样岂不是更要做了,不然多年以后要破产
(D) RenCo's employees are unlikely to have higher cholesterol levels than Garnet's employees——但治疗C还是会存在,即使得病的人少
(E) the average length of time an employee stays with RenCo is less than it is with Garnet——bingo!因为可能发现有胆固醇之前就走了,公司花费只会waste money
7. 1:08
United Lumber will use trees from its forests for two products. The tree trunks will be used for lumber and the branches converted into wood chips to make fiberboard. The cost of this conversion would be the same whether done at the logging site, where the trees are debranched, or at United's factory. However, wood chips occupy less than half the volume of the branches from which they are made.
The information given, if accurate, most strongly supports which of the following?
逻辑链:UL用tree做两种东西,用WC和BRANCH做:FB,不论在不在伐木地区直接坐做FB,成本一样,但是WC占体积是BRANCH的一半
问结论:也就是其实在,体积一半,同样的车材料就能装两倍
这样成本就少了
(A) Converting the branches into wood chips at the logging site would require transporting a fully assembled wood-chipping machine to and from the site.——无关
(B) It would be more economical to debranch the trees at the factory where the fiberboard is manufactured.——反了
(C) The debranching of trees and the conversion of the branches into chips are the only stages in the processing of branches that it would be in United's economic advantage to perform at the logging site.——没有提到是only
(D) Transportation costs from the logging site to the factory that are determined by volume of cargo would be lower if the conversion into chips is done at the logging site rather than at the factory.——bingo!
(E) In the wood-processing industry, branches are used only for the production of wood chips for fiberboard.——没有提到
8. 1:30
Which of the following most logically completes the argument?
Utrania was formerly a major petroleum exporter, but in recent decades economic stagnation and restrictive regulations inhibited investment in new oil fields. In consequence, Utranian oil exports dropped steadily as old fields became depleted. Utrania's currently improving economic situation, together with less-restrictive regulations, will undoubtedly result in the rapid development of new fields. However, it would be premature to conclude that the rapid development of new fields will result in higher oil exports, because __________.
逻辑链:U原来是个石油出口大国,但是最近由于经济衰退和对新油田的投入限制
U的石油出口下降因为老油田也快开发完了
因此U 最近XXXX(干了好多事)可以让石油产量变多
但是过早地认为U石油出口会变高是不对的
问原因:前提,石油产量低,出口低。
现在石油产量高了,出口低
产量高不可质疑,找到可能不会出口的原因,结果必然导致不会高出口
(A) the price of oil is expected to remain relatively stable over the next several years——无关
(B) the improvement in the economic situation in Utrania is expected to result in a dramatic increase in the proportion of Utranians who own automobiles。——bingo!因为内销了
(C) most of the investment in new oil fields in Utrania is expected to come from foreign sources——反了
(D) new technology is available to recover oil from old oil fields formerly regarded as depleted——反了
(E) many of the new oil fields in Utrania are likely to be as productive as those that were developed during the period when Utrania was a major oil exporter——irrelevant
作者: cyndichiang 时间: 2014-4-20 07:09
精练题:43‘’
premise: At most jewelry stores, the person assessing the diamond is the person selling it so you can see why an assessor might say that a diamond is of higher quality than it really is
premise: all diamonds sold at Gem World are certified in writing,
conclusion:you’re assured of a fair price when purchasing a diamond from Gem World.
Q:support
prephraese: you should eliminate the weakening consequence to strengthen the conclusion
if the certified writing is from the selling department, according to the premise1, the real quality is worse than that written in the certified writing.
(A) Many jewelry stores other than Gem World also
provide written certification of the quality of
their diamonds.------irrelevant ,the statement is concerned with the jewelry bought in Gem
(B) The certifications of diamonds at Gem World
are written by people with years of
experience in appraising gems.-------irrelevant, since the argument is judge the accountability of the certified writing, the restate of the conclusion but no explanation of the assessment of the writing is irrelevant
(C) The diamonds sold at Gem World are generally
of higher quality than those sold at other
jewelry stores.------irrelevant, the comparison between the other stores and the Gem World cannot strengthen the argument,since what we should deal with the writing in Gem
(D) The diamond market is so volatile that prices of
the most expensive diamonds can change by
hundreds of dollars from one day to the next.-----opposite,slightly weaken;since the price fluctuates considerably,so the price in the certified writing is not useful to judge the value of jewlry
选项分析错了。答案说 However, the answer choice specifically mentions “the most expensive diamonds” and there is no guarantee that Gem World carries diamonds in this price range. So, at best, the answer choice has no effect on the argument and is therefore incorrect.没有仔细看the most expensive
(E) The written certifications of diamonds at Gem
World are provided by an independent
company of gem specialists.-----Correct,since the statement eliminate a weakening situation, if the certified writing is from the same selling department,according to the premise1, the price cannot reflect the real price
作者: cyndichiang 时间: 2014-4-20 13:45
逻辑链
1. 49‘’
premise:algae grow inside the top layer of sea ice if enough sunlight reaches that layer of ice and enough seawater, which contains nutrients, reaches that layer after splashing onto the surface
conclusion: Even though moderate snow cover reduces the sunlight that filters into the top layer, sea ice with moderate snow cover typically contains even more algae in the top layer than does sea ice with less snow cover.
Q:Support
prephrase: snow cover can increase the number of algae in the top layer by some ways
3. 36''
premise: testing of its employees' cholesterol levels and treatment of high cholesterol saves Garnet money, since high cholesterol left untreated for many years leads to conditions that require very expensive treatment
premise:??
conclusion: RenCo does not have the same financial incentive to adopt such a policy
Q: support
prephrase: 因推果,两个公司的比较,只要指出两个公司的员工在cholesterol的不同点:find the reason why RC does not need the policy
3. 35''
BG:United Lumber will use trees from its forests for two products. The tree trunks will be used for lumber and the branches converted into wood chips to make fiberboard.
premise: The cost of conversion : the logging site, where the trees are debranched = United's factory.
premise: wood chips occupy less than half the volume of the branches .
Conclusion:???
Q:must be true
prephrase: Since chips occupy less volume than the branches,more chips can be transported from logging site to factory than branches thus chips cost less.
4.31''
BG: U was formerly a major petroleum exporter, but in recent decades economic stagnation and restrictive regulations inhibited investment in new oil fields and U's oil exports dropped steadily as old fields became depleted.
premise:U's currently improving economic situation, together with less-restrictive regulations, will undoubtedly result in the rapid development of new fields.
conclusion: it would be premature to conclude that the rapid development of new fields will result in higher oil exports
Q:support
prephrase: new oil fields can be explored and increase the oil production
作者: simonleo 时间: 2014-5-14 13:41
1.
Time:23s
P:In most stores, assesor might say that a diamond is of higher quality than it really is.
Diamonds in Gem World are certified in writing, C:you're assured of a fair price when purchasing a diamond from Gem World
Pre:加强。 说明这个证书别的公司不能自己做啊,总之体现出这个证书的真实有效性。
(A) Many jewelry stores other than Gem World also
provide written certification of the quality of
their diamonds. 不能加强 反而像削弱
(B) The certifications of diamonds at Gem World
are written by people with years of
experience in appraising gems. contender 但是也不好,因为不能说明这些人会按照真实情况来写,也不能说明这些人是来自公司外部
(C) The diamonds sold at Gem World are generally
of higher quality than those sold at other
jewelry stores.无关 不能说明证书有效
(D) The diamond market is so volatile that prices of
the most expensive diamonds can change by
hundreds of dollars from one day to the next.无关
(E) The written certifications of diamonds at Gem
World are provided by an independent
company of gem specialists. 正解,说明其不受公司的影响来判断钻石的品质。
2. 1:25 A
3. 15s E
4. 50s D
5. 1:02 B
作者: 千晤meli 时间: 2015-8-30 11:26
解答了我的疑问,借鉴一下~
E,我的思路是也许去R公司的人已经是生病了的呢,这样就不能省钱了啊。但实际是自己增加了一个假设,不可以再多一个假设。
作者: MIA926 时间: 2015-10-2 22:25
20151002 CR小分队
精练
1. Advertisement: At most jewelry stores, the person
assessing the diamond is the person selling it so
you can see why an assessor might say that a
diamond is of higher quality than it really is.
But because all diamonds sold at Gem World
are certified in writing, you’re assured of a fair
price when purchasing a diamond from Gem
World.
计时:20.70s
逻辑链:珠宝店鉴定宝石的人就是卖宝石的人,所以他们总是把宝石的质量说的比较好。但是因为GW的宝石是有证书的,所以顾客可以确保宝石的价格是fair的
The reasoning in the advertisement would be most
strengthened if which one of the following were true?
(A) Many jewelry stores other than Gem World also
provide written certification of the quality of
their diamonds. 无关
(B) The certifications of diamonds at Gem World
are written by people with years of
experience in appraising gems. 正确
(C) The diamonds sold at Gem World are generally
of higher quality than those sold at other
jewelry stores. 无关
(D) The diamond market is so volatile that prices of
the most expensive diamonds can change by
hundreds of dollars from one day to the next. 无关
(E) The written certifications of diamonds at Gem
World are provided by an independent
company of gem specialists. 削弱
PREP CR 2 逻辑练习
5. (24305-!-item-!-188;#058&001204)
Microscopic plants called algae grow inside the top layer of sea ice in the Antarctic if enough sunlight reaches that layer of ice and enough seawater, which contains nutrients, reaches that layer after splashing onto the surface. Even though moderate snow cover reduces the sunlight that filters into the top layer, sea ice with moderate snow cover typically contains even more algae in the top layer than does sea ice with less snow cover.
推测:要找一个可以解释为什么snow cover可以使得algae增多的选项
Which of the following, if true, most helps to explain the apparent discrepancy?
(A) As the weight of accumulated snow forces ice lower in the water, more seawater splashes onto the surface and seeps through the snow. 正确
(B) Seawater in the Antarctic often contains higher concentrations of nutrients than does seawater in more temperate regions. 无关
(C) As the air temperature around sea ice decreases during the winter, the likelihood decreases that snow will fall and thus add to any existing snow cover. 无关
(D) The nutrients on which algae rely are common in seawater throughout the Antarctic. 无关
(E) More sunlight filters through a layer of ice than through an equally thick layer of snow.
6. (24353-!-item-!-188;#058&001207)
Which of the following most logically completes the passage?
Garnet and RenCo each provide health care for their employees. Garnet pays for both testing of its employees' cholesterol levels and treatment of high cholesterol. This policy saves Garnet money, since high cholesterol left untreated for many years leads to conditions that require very expensive treatment. However, RenCo does not have the same financial incentive to adopt such a policy, because __________.
(A) early treatment of high cholesterol does not entirely eliminate the possibility of a stroke later in life 错误 不完全消除但是还是比不消除好
(B) the mass media regularly feature stories encouraging people to maintain diets that are low in cholesterol 无关
(C) RenCo has significantly more employees than Garnet has 削弱
(D) RenCo's employees are unlikely to have higher cholesterol levels than Garnet's employees比较无关
(E) the average length of time an employee stays with RenCo is less than it is with Garnet 正确
7. (24401-!-item-!-188;#058&001291)
United Lumber will use trees from its forests for two products. The tree trunks will be used for lumber and the branches converted into wood chips to make fiberboard. The cost of this conversion would be the same whether done at the logging site, where the trees are debranched, or at United's factory. However, wood chips occupy less than half the volume of the branches from which they are made.
The information given, if accurate, most strongly supports which of the following?
(A) Converting the branches into wood chips at the logging site would require transporting a fully assembled wood-chipping machine to and from the site. 新消息
(B) It would be more economical to debranch the trees at the factory where the fiberboard is manufactured. 正确
(C) The debranching of trees and the conversion of the branches into chips are the only stages in the processing of branches that it would be in United's economic advantage to perform at the logging site. 新消息
(D) Transportation costs from the logging site to the factory that are determined by volume of cargo would be lower if the conversion into chips is done at the logging site rather than at the factory. 反信息
(E) In the wood-processing industry, branches are used only for the production of wood chips for fiberboard.未知
8. (24449-!-item-!-188;#058&001338)
Which of the following most logically completes the argument?
Utrania was formerly a major petroleum exporter, but in recent decades economic stagnation and restrictive regulations inhibited investment in new oil fields. In consequence, Utranian oil exports dropped steadily as old fields became depleted. Utrania's currently improving economic situation, together with less-restrictive regulations, will undoubtedly result in the rapid development of new fields. However, it would be premature to conclude that the rapid development of new fields will result in higher oil exports, because __________.
推测:要找一个能说明为什么有新的开采点但是出口不一定会增加的选项
(A) the price of oil is expected to remain relatively stable over the next several years 无关
(B) the improvement in the economic situation in Utrania is expected to result in a dramatic increase in the proportion of Utranians who own automobiles 正确 汽车多了 国内用油量增加
(C) most of the investment in new oil fields in Utrania is expected to come from foreign sources
(D) new technology is available to recover oil from old oil fields formerly regarded as depleted 削弱
(E) many of the new oil fields in Utrania are likely to be as productive as those that were developed during the period when Utrania was a major oil exporter 削弱
作者: fesche 时间: 2016-5-21 03:08
person accessing is the selling
person say its high quality
GW, certificate
assure fair price
strengthen
因:有certificate的
果:价格公道
E
A water sunlight
more layer of snow more A than no snow
解释,需要阳光和海水
一层雪会遮挡阳光,但是落雪的会比没雪的多A
海水更多养分
A
(A) As the weight of accumulated snow forces ice lower in the water, more seawater splashes onto the surface and seeps through the snow.
(B) Seawater in the Antarctic often contains higher concentrations of nutrients than does seawater in more temperate regions.
(C) As the air temperature around sea ice decreases during the winter, the likelihood decreases that snow will fall and thus add to any existing snow cover.
(D) The nutrients on which algae rely are common in seawater throughout the Antarctic.
(E) More sunlight filters through a layer of ice than through an equally thick layer of snow.
E
lumber, branches for fire board
can be done in debranches or in factory
wood chips less than half of what the company made
哪个是对的,
United Lumber will use trees from its forests for two products. The tree trunks will be used for lumber and the branches converted into wood chips to make fiberboard. The cost of this conversion would be the same whether done at the logging site, where the trees are debranched, or at United's factory. However, wood chips occupy less than half the volume of the branches from which they are made.
树身拿来做lumber,树枝做WC然后做fiber
价格在两个地方都一样
WC比树枝少一半
C,BCD都可以,B和economical应该没有关系
(A) Converting the branches into wood chips at the logging site would require transporting a fully assembled wood-chipping machine to and from the site.
(B) It would be more economical to debranch the trees at the factory where the fiberboard is manufactured.
(C) The debranching of trees and the conversion of the branches into chips are the only stages in the processing of branches that it would be in United's economic advantage to perform at the logging site.
(D) Transportation costs from the logging site to the factory that are determined by volume of cargo would be lower if the conversion into chips is done at the logging site rather than at the factory.
(E) In the wood-processing industry, branches are used only for the production of wood chips for fiberboard.
B
作者: May97 时间: 2018-3-7 09:14
T: 29S
P: the person assessing the diamond is the person selling it so you can see why an assessor might say that a diamond is of higher quality than it really is.
p: But because all diamonds sold at Gem World are certified in writing,
C: you’re assured of a fair price when purchasing a diamond from Gem World.
SUPPORT: CERTIFIED IN WRITING降低assessor高估价格的可能性
(A) Many jewelry stores other than Gem World also provide written certification of the quality of their diamonds.--->IRREVERENT
(B) The certifications of diamonds at Gem World are written by people with years of experience in appraising gems.--->IRREVERENT
(C) The diamonds sold at Gem World are generally of higher quality than those sold at other jewelry stores.--->IRREVERENT
(D) The diamond market is so volatile that prices of the most expensive diamonds can change by hundreds of dollars from one day to the next.--->IRREVERENT
(E) The written certifications of diamonds at Gem World are provided by an independent company of gem specialists.--->R
T: 42S
P: Microscopic plants grow if enough sunlight and enough seawater reaches that layer
P: enough seawater after splashing onto the surface.
P: moderate snow cover reduces the sunlight that filters into the top layer,
C: sea ice with moderate snow cover typically contains even more algae in the top layer than does sea ice with less snow cover.
SUPPORT: SEA ICE WITH MODERATE SNOW HAS MORE SEAWATER
(A) As the weight of accumulated snow forces ice lower in the water, more seawater splashes onto the surface and seeps through the snow.--->R
(B) Seawater in the Antarctic often contains higher concentrations of nutrients than does seawater in more temperate regions.--->IRREVERENT
(C) As the air temperature around sea ice decreases during the winter, the likelihood decreases that snow will fall and thus add to any existing snow cover.--->IRREVERENT
(D) The nutrients on which algae rely are common in seawater throughout the Antarctic.--->IRREVERENT
(E) More sunlight filters through a layer of ice than through an equally thick layer of snow.--->WEAKEN
T: 31S
BG: Garnet and RenCo each provide health care for their employees. Garnet pays for both testing of its employees' cholesterol levels and treatment of high cholesterol.
P: This policy saves Garnet money, since high cholesterol left untreated for many years leads to conditions that require very expensive treatment.
C: However, RenCo does not have the same financial incentive to adopt such a policy
SUPPORT: TESTING OF CHOLESTEROL NEEDS MORE MONEY
(A) early treatment of high cholesterol does not entirely eliminate the possibility of a stroke later in life--->NO NEED TO ELIMINATE
(B) the mass media regularly feature stories encouraging people to maintain diets that are low in cholesterol--->IRREVERENT
(C) RenCo has significantly more employees than Garnet has--->IRREVERENT
(D) RenCo's employees are unlikely to have higher cholesterol levels than Garnet's employees--->IRREVERENT
(E) the average length of time an employee stays with RenCo is less than it is with Garnet--->R
T: 61S
BG: United Lumber will use trees from its forests for two products. The tree trunks will be used for lumber and the branches converted into wood chips to make fiberboard.
P: The cost of this conversion would be the same whether done at the logging site
P: However, wood chips occupy less than half the volume of the branches from which they are made.
CONCLUSION: COST和why more branches don't go into wood chips之间的关系
(A) Converting the branches into wood chips at the logging site would require transporting a fully assembled wood-chipping machine to and from the site.--->NEW INFORMATION
(B) It would be more economical to debranch the trees at the factory where the fiberboard is manufactured.--->R
(C) The debranching of trees and the conversion of the branches into chips are the only stages in the processing of branches that it would be in United's economic advantage to perform at the logging site--->IRREVERENT
(D) Transportation costs from the logging site to the factory that are determined by volume of cargo would be lower if the conversion into chips is done at the logging site rather than at the factory.--->IRREVERENT--->R
(E) In the wood-processing industry, branches are used only for the production of wood chips for fiberboard.--->WRONG
这题一脸懵逼。。。
T: 43S
BG: Utrania was formerly a major petroleum exporter, but in recent decades economic stagnation and restrictive regulations inhibited investment in new oil fields. In consequence, Utranian oil exports dropped steadily as old fields became depleted.
P: Utrania's currently improving economic situation, together with less-restrictive regulations, will undoubtedly result in the rapid development of new fields.
C: However, it would be premature to conclude that the rapid development of new fields will result in higher oil exports
SUPPORT: MORE OIL IS CONSUMED in domestic
(A) the price of oil is expected to remain relatively stable over the next several years--->IRREVERENT
(B) the improvement in the economic situation in Utrania is expected to result in a dramatic increase in the proportion of Utranians who own automobiles--->R
(C) most of the investment in new oil fields in Utrania is expected to come from foreign sources--->IRREVERENT
(D) new technology is available to recover oil from old oil fields formerly regarded as depleted--->IRREVERENT
(E) many of the new oil fields in Utrania are likely to be as productive as those that were developed during the period when Utrania was a major oil exporter--->IRREVERENT
作者: echo-LUO 时间: 2018-10-1 22:49
十一快乐!!玩了一整天...罪恶,明天又开始杀G之旅
精练
给钻石股价的人就是卖钻石的人——>估价大于实际价值——>但GW有书面认证——>价格还是合理的
(A) Many jewelry stores other than Gem World also provide written certification of the quality of their diamonds. 无关
(B) The certifications of diamonds at Gem World are written by people with years of experience in appraising gems. = =emmmm
(C) The diamonds sold at Gem World are generally of higher quality than those sold at other jewelry stores. 质量...说的是价格,无关
(D) The diamond market is so volatile that prices of the most expensive diamonds can change by hundreds of dollars from one day to the next. 削弱
(E) The written certifications of diamonds at Gem World are provided by an independent company of gem specialists. !可以,第三方
作者: 云栈 时间: 2019-11-11 09:15
1. 25s
P: 在大多数珠宝店中,评估钻石的人就是出售钻石的人,因此您可以看到为什么评估者会说钻石的质量比实际更高。
C:但是,由于在Gem World上出售的所有钻石都是经过书面认证的,因此从Gem World购买钻石时,您可以放心价格公道。
推测:写certify的人不是卖钻石的人;写certify的人很专业
选项分析:E
A: 其他店是否提供证书与GW的钻石真实性无关
B:写证书的人就算很专业 也有可能故意写的很高
C: 质量与真实性无关
D: opposite
E: 独立说明他们和卖钻石的没关系,所以公平
2. 40s
P:如果有足够的阳光照射到南极洲的冰层中,并且有足够的海水(其中含有营养物质)在溅到地表之后到达该层,那么称为南极的微观植物就会生长在南极海冰的顶层。
C: 即使适度的积雪减少了过滤到顶层的阳光,但具有适度积雪的海冰通常在顶层包含的藻类比具有较少积雪的海冰的藻类还要多。
推测:snow cover的地方的水含的营养更多,弥补了没有阳光的缺陷
选项分析:A
【这题错了,如果证明阳光变少了,就要说明另一个条件,海水变多了;B选项说的是南极洲的海水营养比别的地方丰富,但是并不代表积雪多的地方就更丰富】
3. 34s
P: G和R各自为员工提供医疗保健。G既用于测试员工的胆固醇水平,又用于治疗高胆固醇。
P: 这项政策可以为石榴石省钱,因为多年未治疗的高胆固醇会导致需要非常昂贵治疗的疾病。
C: 但是,RenCo没有采用相同政策的经济动机,因为__________。
推测:签了他们公司的人如果得了高胆固醇需要自费治疗;公司员工都是待短期的
选项分析:E
4. 40s
P: 联合木材公司将使用其森林中的树木作为两种产品。树干将被用作木材,而树枝则被转变成木屑以制成纤维板。
P: 无论是在伐木现场(树木被分枝)还是在美联航的工厂进行,这种转换的成本都是相同的。
P: 但是,木屑所占比例不到制造树枝的一半。
推测: 推不出来
选项分析: D
5. 44s
P:Utrania以前是主要的石油出口国,但近几十年来,经济停滞和限制性法规抑制了对新油田的投资。
P: 结果,随着老油田的枯竭,乌特兰人的石油出口稳步下降。
P: 乌特兰尼亚目前改善的经济状况,加上限制性较小的法规,无疑将导致新领域的快速发展。
C: 但是,现在断定新油田的快速发展将导致更高的石油出口量为时尚早,因为__________。
推测:别的原来进口石油的国家可能也develop new field,从而降低了石油进口需求量
选项分析:B
作者: Lincy123 时间: 2020-5-29 08:48
C:在Gem world的钻石都有证书
P:在Gem world买钻石确保有公允价格
Gap:证书=公允价格
A) 许多小店也提供证书。Weaken
B) Gem world的证书是由多年评估经验的人写的,无关
C) 在gem world卖掉的钻石的质量普遍比其他商店高,无关
D)钻石市场十分动荡,价格隔一天就可以查几百美元,无关
E) 证书是由独立机构的钻石专家提供的。Correct
尽管moderate雪减少了进入顶层的阳光,被m雪覆盖的海冰比更少雪的海冰由更多顶层的藻类。
Gap:moderate snow – algae
A) 因为雪的重量把冰压低,更多海水进入顶层,correct
B) 在A的海水比其他海水更有营养,与snow无关
C) 冬天温度下降,雪的减少机率下降,无关
D)藻类需要的营养在整个antarctic都有,无关
E) 同样厚的冰和雪,阳光更多通过冰
G为员工检查和治疗高血压花钱,政策给G优惠因为心血管治疗很贵。R没有,因为
这个政策省钱,因为长期没被治疗的高胆固醇患者需要花更多钱治病。R没有因为这个省钱动因去采取同样的政策,因为
A) 早期治疗不能消除后期中风风险。无关
B) 媒体鼓励人们健康饮食。无关
C) R比G员工多。无关
D)R员工比G员工更不可能有高血压,correct(x)
R员工的胆固醇水平比G员工低,原文并未提到胆固醇高低会有何种影响
E) R员工的平均留存时间比G少。Correct
R员工既然时间短,长期未治疗的人的可能性就比较低,如果采用这个政策反而亏钱
L和B被做成WC再做成F。是否在伐木场制作,成本都一样。但是WC的体积比原材料B少了不止一倍。
A) 在伐木场转换需要把机器来回运输,无关
B) 在工厂制造会更节省。No
C) 把b转换成chip是唯一流程,在伐木场更便宜。无关
D)根据体积计算的运输费用会更少,correct
E) 在制造工厂,B只用来生产WC,weaken
U现在改善方法就是开发新油田,但是快速开发新油田会导致更高的油出口这个结论下的太早,因为?
A) 油价预期在接下来几年保持不变,无关
B) U的经济增长预期会在拥有汽车的U市民的比例增长上。Correct
有车的市民多了,国内的油需求就多,出口不一定多
C) 大部分新油田的投资预期是国外的资源。Correct(x)无关
D)恢复旧油田的新技术可得。无关
E) 新油田的产量和U是油出口大亨时差不多,无关
欢迎光临 ChaseDream (https://forum.chasedream.com/) |
Powered by Discuz! X3.3 |